Dictionary of Arguments


Philosophical and Scientific Issues in Dispute
 
[german]

Screenshot Tabelle Begriffes

 

Find counter arguments by entering NameVs… or …VsName.

Enhanced Search:
Search term 1: Author or Term Search term 2: Author or Term


together with


The author or concept searched is found in the following 25 controversies.
Disputed term/author/ism Author Vs Author
Entry
Reference
Behaviorism Searle Vs Behaviorism I 30
Searle: the ontology of mental states is an ontology of the first person. (SearleVsBehavoirism).
I 49
SearleVsBehaviorism: two types of objections: 1. objections of common sense. 2. Technical objections. 1. VsLogical Behaviorism: technical Objections: behaviorism never succeeded to fully explain the concept of a "disposition".
Circle: if one wants to analyze the belief through behavior then you have to obviously also make reference to the wishing; if one wants to analyze the wishing by behavior, then you have obviously also have to make reference to the belief (Chisholm 1957)(1).
I 50
LewisVsBehaviorism: technical objection: behaviorism ignores the causal relationships between mental states and behavior (Lewis 1966)(2). The Objections of common sense are ultimately the most embarrassing. The absurdity lies in the denial of the existence of all the inner states of mind. This is against our ordinary experience of how it is to be a human being.
I 57
Functionalism: what makes two neurophysiological states relating to occurrences of the same state of mind type, is that they perform the same function throughout the life of the organism. The two mind states must then stand on the following three things in the same causal relations: 1. To the stimulus that the organism receives as input,
2. To the various other "mental" states and
3. To the behavior that the organism produces as output.
Note that by the causal relationships two Objections are avoided that were put forward VsBehaviorism: the first said that behaviorism neglects the causal relationship of mental states, the other said that in it a circularity was contained, and as convictions against recourse to requests and wishes had to be analyzed by resorting to convictions.
VIII 428
Grammar/language/SearleVsBehaviorism/SearleVsEmpiricism: Dilemma:
a) Either he relies solely on stimulus-response mechanisms (stimulus response) then he can not explain the acquisition of Grammar. Or
b) He admits à la Quine that there are innate mechanisms. But once the mechanisms are rich enough, the stimulus-response part is not interesting!


1. R. Chisholm, Perceiving Ithaca, NY, 1957
2. D. Lewis, An argument for the identity theory, Journal of Philosophy 63, 1966: pp.17-25

Searle I
John R. Searle
The Rediscovery of the Mind, Massachusetts Institute of Technology 1992
German Edition:
Die Wiederentdeckung des Geistes Frankfurt 1996

Searle II
John R. Searle
Intentionality. An essay in the philosophy of mind, Cambridge/MA 1983
German Edition:
Intentionalität Frankfurt 1991

Searle III
John R. Searle
The Construction of Social Reality, New York 1995
German Edition:
Die Konstruktion der gesellschaftlichen Wirklichkeit Hamburg 1997

Searle IV
John R. Searle
Expression and Meaning. Studies in the Theory of Speech Acts, Cambridge/MA 1979
German Edition:
Ausdruck und Bedeutung Frankfurt 1982

Searle V
John R. Searle
Speech Acts, Cambridge/MA 1969
German Edition:
Sprechakte Frankfurt 1983

Searle VII
John R. Searle
Behauptungen und Abweichungen
In
Linguistik und Philosophie, G. Grewendorf/G. Meggle Frankfurt/M. 1974/1995

Searle VIII
John R. Searle
Chomskys Revolution in der Linguistik
In
Linguistik und Philosophie, G. Grewendorf/G. Meggle Frankfurt/M. 1974/1995

Searle IX
John R. Searle
"Animal Minds", in: Midwest Studies in Philosophy 19 (1994) pp. 206-219
In
Der Geist der Tiere, D Perler/M. Wild Frankfurt/M. 2005
Bühler, K. Quine Vs Bühler, K. VII (c) 50/51
Grammar/Quine: objective problem: every speech act is considered an element of K. But to distinguish the individual elements, i.e. to group them into similar acoustic events, he must collect them into bundles of specific density, which qualify them as linguistic (language-related) forms. For this he needs the concept of the phoneme as generally relative term.
Thus Grammar is based on the two terms "significant" and "phoneme".
How can we free Grammar and the concept of the phoneme itself from the concept of synonymy?
Solution/Bühler, Karl: the continuum of acoustic events can be represented in a three-dimensional diagram in which the frequency of occurrence is taken into account. The biggest bulges would then be the phonemes.
QuineVsBühler: there are plenty of reasons to consider neither this over-simplified picture nor anything else a definition of "phoneme".

Quine I
W.V.O. Quine
Word and Object, Cambridge/MA 1960
German Edition:
Wort und Gegenstand Stuttgart 1980

Quine II
W.V.O. Quine
Theories and Things, Cambridge/MA 1986
German Edition:
Theorien und Dinge Frankfurt 1985

Quine III
W.V.O. Quine
Methods of Logic, 4th edition Cambridge/MA 1982
German Edition:
Grundzüge der Logik Frankfurt 1978

Quine V
W.V.O. Quine
The Roots of Reference, La Salle/Illinois 1974
German Edition:
Die Wurzeln der Referenz Frankfurt 1989

Quine VI
W.V.O. Quine
Pursuit of Truth, Cambridge/MA 1992
German Edition:
Unterwegs zur Wahrheit Paderborn 1995

Quine VII
W.V.O. Quine
From a logical point of view Cambridge, Mass. 1953

Quine VII (a)
W. V. A. Quine
On what there is
In
From a Logical Point of View, Cambridge, MA 1953

Quine VII (b)
W. V. A. Quine
Two dogmas of empiricism
In
From a Logical Point of View, Cambridge, MA 1953

Quine VII (c)
W. V. A. Quine
The problem of meaning in linguistics
In
From a Logical Point of View, Cambridge, MA 1953

Quine VII (d)
W. V. A. Quine
Identity, ostension and hypostasis
In
From a Logical Point of View, Cambridge, MA 1953

Quine VII (e)
W. V. A. Quine
New foundations for mathematical logic
In
From a Logical Point of View, Cambridge, MA 1953

Quine VII (f)
W. V. A. Quine
Logic and the reification of universals
In
From a Logical Point of View, Cambridge, MA 1953

Quine VII (g)
W. V. A. Quine
Notes on the theory of reference
In
From a Logical Point of View, Cambridge, MA 1953

Quine VII (h)
W. V. A. Quine
Reference and modality
In
From a Logical Point of View, Cambridge, MA 1953

Quine VII (i)
W. V. A. Quine
Meaning and existential inference
In
From a Logical Point of View, Cambridge, MA 1953

Quine VIII
W.V.O. Quine
Designation and Existence, in: The Journal of Philosophy 36 (1939)
German Edition:
Bezeichnung und Referenz
In
Zur Philosophie der idealen Sprache, J. Sinnreich (Hg) München 1982

Quine IX
W.V.O. Quine
Set Theory and its Logic, Cambridge/MA 1963
German Edition:
Mengenlehre und ihre Logik Wiesbaden 1967

Quine X
W.V.O. Quine
The Philosophy of Logic, Cambridge/MA 1970, 1986
German Edition:
Philosophie der Logik Bamberg 2005

Quine XII
W.V.O. Quine
Ontological Relativity and Other Essays, New York 1969
German Edition:
Ontologische Relativität Frankfurt 2003

Quine XIII
Willard Van Orman Quine
Quiddities Cambridge/London 1987
Chomsky, N. Pinker Vs Chomsky, N. Dennett I 545/546
Steven PinkerVsChomsky: specialization to the grammar is a conventional neo-Darwinist process. The majority of the most interesting properties of the "language organ" must have evolved through adaptation.   Pinker: the Objections to this position are mostly ridiculous - e.g. the structure of the cell should be "purely physical" and explained without evolution - e.g. language were not designed to communicate, etc.

Pinker I 218
Design/Chomsky: It is wrong to make selection responsible for all design: E.g. the fact that I have a positive mass prevents me from eloping into outer space, but has nothing to do with selection. Simple physical explanation. Explanation/Selection/PinkerVsChomsky: you usually do not refer to selection to explain utility, but to explain something improbable. E.g. eye. If we calculate the parts of the universe with a positive mass and those equipped with an eye, we need an explanation for this difference. Vs: one might reply: the criterion: seeing/not-seeing was only introduced in retrospect, after we knew what animals are capable of. I 219 Most clusters of matter cannot see, but most cannot "fle" either, and I define that now as the composition, size and shape of the stone, on which I'm sitting now.
Def Design/Pinker: If the function cannot be described more economically than the structure, no design is present. The concept of function adds nothing new.
Design/Pinker: should not serve the harmony of the ecosystem or the beauty of nature. After all, the replicator must be the beneficiary.

Pi I
St. Pinker
How the Mind Works, New York 1997
German Edition:
Wie das Denken im Kopf entsteht München 1998

Dennett I
D. Dennett
Darwin’s Dangerous Idea, New York 1995
German Edition:
Darwins gefährliches Erbe Hamburg 1997

Dennett II
D. Dennett
Kinds of Minds, New York 1996
German Edition:
Spielarten des Geistes Gütersloh 1999

Dennett III
Daniel Dennett
"COG: Steps towards consciousness in robots"
In
Bewusstein, Thomas Metzinger Paderborn/München/Wien/Zürich 1996

Dennett IV
Daniel Dennett
"Animal Consciousness. What Matters and Why?", in: D. C. Dennett, Brainchildren. Essays on Designing Minds, Cambridge/MA 1998, pp. 337-350
In
Der Geist der Tiere, D Perler/M. Wild Frankfurt/M. 2005
Chomsky, N. Putnam Vs Chomsky, N. Chomsky I 293
PutnamVsChomsky: Putnam assumes for phonetics in the universal grammar, that it only has a single list of sounds. This did not require a sophisticated explanatory hypothesis. Only "memory span and powers of recollection". "No upright behaviorist would deny that these are innate properties." ChomskyVsPutnam: but there have been set up very strong empirical hypotheses about the selection of the universal distinctive features, none of which seems to be explained on the basis of restrictions of memory.
Chomsky I 298
PutnamVsChomsky: Thesis: instead of an innate schematism, "general multipurpose strategies" could be assumed. This innate base would have to be the same for the acquisition of any knowledge, so that there is nothing special about language acquisition.
Chomsky I 299
ChomskyVsPutnam: with that he is no longer entitled to assume something is innate. Furthermore, it only shifts the problem. PutnamVsChomsky: the evaluation functions proposed in the universal Grammar "the kind of facts is constituted which tries to explain the theory of learning, but not the required explanation itself".
ChomskyVsPutnam: E.g. no one would say that the genetic basis for the development of arms instead of wings was "the kind of fact that attempts to explain the theory of learning". Rather, they are the basis for an explanation of other facts of human behavior.
Whether the evaluation function is learned or is the basis of learning, is an empirical question.
PutnamVsChomsky: certain ambiguities can only be discovered by routine, therefore their postulated explanation by Chomsky's Grammar is not very impressive.
ChomskyVsPutnam: he misunderstands it, in fact that refers to competence and not to performance (actual practice).
What the Grammar explains is why e.g. in "criticism of students" "student" can be understood as subject or Object, whereas e.g. "grain" in "the growing of the grain" can only be subject.
The question of routine does not matter here.
Chomsky I 300
Innate Ideas/ChomskyVsPutnam: the innate representation of universal grammar indeed solves the problem of learning (at least partly) if it is really true that this is the basis for language acquisition, which may very well be the case!
Putnam III 87
Putnam/Chomsky: Putnam proposes: correctness in linguistics is what the currently available data best explain about the behavior of the speaker under a current interest. What is true today, will be false tomorrow. PutnamVsChomsky: I never said that what is right today, will be wrong tomorrow.
Putnam: Chomsky's hidden main theses:
1) the we are free to choose our interests at will,
2) that interests themselves are not subject to normative criticism.
E.g. Hans' heart attack lies in the defiance of medical recommendations. Other explanation: high blood pressure. It may be, in fact, that on one day one fact is more in the interests of the speaker, and the next day another one.
III 88
PutnamVsChomsky: 1) we cannot just pick and choose our interests. 2) It sometimes happens that the relevance of a particular interest is disputed. How can it be, however, that some interests are more reasonable than others? Reasonableness is supposed to depend on different conditions in different contexts. There is no general answer.
III 88/89
The assertion that a concept is interest relative does not come out at the same as the thesis, all interests are equally reasonable.

Putnam I
Hilary Putnam
Von einem Realistischen Standpunkt
In
Von einem realistischen Standpunkt, Vincent C. Müller Frankfurt 1993

Putnam I (a)
Hilary Putnam
Explanation and Reference, In: Glenn Pearce & Patrick Maynard (eds.), Conceptual Change. D. Reidel. pp. 196--214 (1973)
In
Von einem realistischen Standpunkt, Vincent C. Müller Reinbek 1993

Putnam I (b)
Hilary Putnam
Language and Reality, in: Mind, Language and Reality: Philosophical Papers, Volume 2. Cambridge University Press. pp. 272-90 (1995
In
Von einem realistischen Standpunkt, Vincent C. Müller Reinbek 1993

Putnam I (c)
Hilary Putnam
What is Realism? in: Proceedings of the Aristotelian Society 76 (1975):pp. 177 - 194.
In
Von einem realistischen Standpunkt, Vincent C. Müller Reinbek 1993

Putnam I (d)
Hilary Putnam
Models and Reality, Journal of Symbolic Logic 45 (3), 1980:pp. 464-482.
In
Von einem realistischen Standpunkt, Vincent C. Müller Reinbek 1993

Putnam I (e)
Hilary Putnam
Reference and Truth
In
Von einem realistischen Standpunkt, Vincent C. Müller Reinbek 1993

Putnam I (f)
Hilary Putnam
How to Be an Internal Realist and a Transcendental Idealist (at the Same Time) in: R. Haller/W. Grassl (eds): Sprache, Logik und Philosophie, Akten des 4. Internationalen Wittgenstein-Symposiums, 1979
In
Von einem realistischen Standpunkt, Vincent C. Müller Reinbek 1993

Putnam I (g)
Hilary Putnam
Why there isn’t a ready-made world, Synthese 51 (2):205--228 (1982)
In
Von einem realistischen Standpunkt, Vincent C. Müller Reinbek 1993

Putnam I (h)
Hilary Putnam
Pourqui les Philosophes? in: A: Jacob (ed.) L’Encyclopédie PHilosophieque Universelle, Paris 1986
In
Von einem realistischen Standpunkt, Vincent C. Müller Reinbek 1993

Putnam I (i)
Hilary Putnam
Realism with a Human Face, Cambridge/MA 1990
In
Von einem realistischen Standpunkt, Vincent C. Müller Reinbek 1993

Putnam I (k)
Hilary Putnam
"Irrealism and Deconstruction", 6. Giford Lecture, St. Andrews 1990, in: H. Putnam, Renewing Philosophy (The Gifford Lectures), Cambridge/MA 1992, pp. 108-133
In
Von einem realistischen Standpunkt, Vincent C. Müller Reinbek 1993

Putnam II
Hilary Putnam
Representation and Reality, Cambridge/MA 1988
German Edition:
Repräsentation und Realität Frankfurt 1999

Putnam III
Hilary Putnam
Renewing Philosophy (The Gifford Lectures), Cambridge/MA 1992
German Edition:
Für eine Erneuerung der Philosophie Stuttgart 1997

Putnam IV
Hilary Putnam
"Minds and Machines", in: Sidney Hook (ed.) Dimensions of Mind, New York 1960, pp. 138-164
In
Künstliche Intelligenz, Walther Ch. Zimmerli/Stefan Wolf Stuttgart 1994

Putnam V
Hilary Putnam
Reason, Truth and History, Cambridge/MA 1981
German Edition:
Vernunft, Wahrheit und Geschichte Frankfurt 1990

Putnam VI
Hilary Putnam
"Realism and Reason", Proceedings of the American Philosophical Association (1976) pp. 483-98
In
Truth and Meaning, Paul Horwich Aldershot 1994

Putnam VII
Hilary Putnam
"A Defense of Internal Realism" in: James Conant (ed.)Realism with a Human Face, Cambridge/MA 1990 pp. 30-43
In
Theories of Truth, Paul Horwich Aldershot 1994

SocPut I
Robert D. Putnam
Bowling Alone: The Collapse and Revival of American Community New York 2000

Chomsky I
Noam Chomsky
"Linguistics and Philosophy", in: Language and Philosophy, (Ed) Sidney Hook New York 1969 pp. 51-94
In
Linguistik und Philosophie, G. Grewendorf/G. Meggle Frankfurt/M. 1974/1995

Chomsky II
Noam Chomsky
"Some empirical assumptions in modern philosophy of language" in: Philosophy, Science, and Method, Essays in Honor of E. Nagel (Eds. S. Morgenbesser, P. Suppes and M- White) New York 1969, pp. 260-285
In
Linguistik und Philosophie, G. Grewendorf/G. Meggle Frankfurt/M. 1974/1995

Chomsky IV
N. Chomsky
Aspects of the Theory of Syntax, Cambridge/MA 1965
German Edition:
Aspekte der Syntaxtheorie Frankfurt 1978

Chomsky V
N. Chomsky
Language and Mind Cambridge 2006
Chomsky, N. Strawson Vs Chomsky, N. VI 386
Transformational grammar: two kinds of formative: 1. lexical: correspond to names and general terms whose meaning is not somehow derived syntactically: e.g. "to sing", "to love", "red", "Mary".
2. non-lexical: heterogeneous group: e.g. the formative "pret" for the past tense.
There is no mechanical process to find the deep structure.
VI 389
Thesis: "Unconscious mastery" or "internal representation" is not enough to explain the linguistic abilities. The rules of transformational Grammar provide the basis for the determination of those grammatical relations which are decisive for the semantic interpretation of sentences though not alone determining.
VI 390
Grammar not circular, because it contains a lexicon. StrawsonVsChomsky: there is no general theory of the decisive class of compounds (of grammatical categories and formatives).
VI 391
There is only the list of items in the dictionary without any representation of general principles of the allocation. But we should expect just such a theory if the Grammar is to satisfy the conditions of transparency.
Because we define with the grammatical categories the functions and relations of the sentence elements. That is what everyone understands without having explicitly learned Grammar. We combine obvious semantic and syntactic considerations.
VI 392
Explanation/Chomsky: this one admits that a "descriptively adequate" grammar must not be "explanation adequate". We need a theory of linguistic universals.
In addition, it must be explained how our Grammar was selected from other possible Grammars.
It must be explained:
1. Why do we understand infinitely many new propositions? (> See also the discussion "Is Language infinite?"). 2. The connection of semantics and syntax.
VI 393
StrawsonVsChomsky: comments only expressly reserved on semantic considerations. Dictionary/Chomsky: is part of the base and contains far fewer entries than our ordinary dictionary.
VI 395
Transformation grammar Vs traditional grammar: it was too unsystematic, no explanation of the traditional terms "verb", "noun", "object" possible.
VI 396
PhilosophyVsGrammar/Strawson: is first freed from "empirical" requirements, does initially not need to cope with the actual formal requirements He has just like the Grammarian a conception of meaning elements and a conception of semantically significant combination modes of these elements, to which the vocabulary is available in a transparent relationship.
With these transparent relations he can consider possible formal arrangements by whom the combining functions could be dispensed.
This is reminiscent of the construction of ideal languages.
VI 397
Quine: (anywhere): "Do not show more structure than necessary". Grammar/Strawson: one must always distinguish between the actual (essential, crucial) and possible Grammars.
E.g. the essential Grammar must show what elements belong to which, all combinations must be shown and be possible to distinguish.
E.g. it must be possible to show when an element describes a non-symmetric relation.
But the essential Grammar determines in no way how these requirements are to be fulfilled.
VI 398
We can choose one of several grammars. If it fulfills the requirements, we have a complete and totally transparent grammar. (Only idealized simplified, that is the price). Vocabulary/Strawson: we need a completely elaborate vocabulary or a set of related vocabularies.
1. Ontological vocabulary e.g. space, time, thing, gen. characteristics
2. Semantic V., for types and individual (abstract) elements, proper names for things,
3. Functional V. for combination or relation types. Deictic elements.
4. Vocabulary of the formal apparatus.

Strawson I
Peter F. Strawson
Individuals: An Essay in Descriptive Metaphysics. London 1959
German Edition:
Einzelding und logisches Subjekt Stuttgart 1972

Strawson II
Peter F. Strawson
"Truth", Proceedings of the Aristotelian Society, Suppl. Vol XXIV, 1950 - dt. P. F. Strawson, "Wahrheit",
In
Wahrheitstheorien, Gunnar Skirbekk Frankfurt/M. 1977

Strawson III
Peter F. Strawson
"On Understanding the Structure of One’s Language"
In
Truth and Meaning, G. Evans/J. McDowell Oxford 1976

Strawson IV
Peter F. Strawson
Analysis and Metaphysics. An Introduction to Philosophy, Oxford 1992
German Edition:
Analyse und Metaphysik München 1994

Strawson V
P.F. Strawson
The Bounds of Sense: An Essay on Kant’s Critique of Pure Reason. London 1966
German Edition:
Die Grenzen des Sinns Frankfurt 1981

Strawson VI
Peter F Strawson
Grammar and Philosophy in: Proceedings of the Aristotelian Society, Vol 70, 1969/70 pp. 1-20
In
Linguistik und Philosophie, G. Grewendorf/G. Meggle Frankfurt/M. 1974/1995

Strawson VII
Peter F Strawson
"On Referring", in: Mind 59 (1950)
In
Eigennamen, Ursula Wolf Frankfurt/M. 1993
Cognition Theory Searle Vs Cognition Theory I 8
SearleVsCognition Psychology: Black Box, the cognitive scientists repeat the worst mistake of the behaviorists: they insisted to examine only Objectively observable phenomena. So they left the essential characteristics of the mind aside. In the big black box they only found a lot of small black boxes.
I 217
cognition theory: Here it is claimed, we would have drawn a conclusion, when we look at a tree from one and then know that he has a back. SearleVsCognition theory: On the contrary, what we do is simply this: we see a tree as a real tree. The background is not a control system.
I 222
SearleVs Cognitive Science/VsCognition: the basic assumptions of cognitive science are wrong. Cognitive Science: neither the examination of the brain nor the study of consciousness is of interest or value.
Although the cognitive mechanisms are actually in the brain, and some of them refer to the awareness a superficial expression, but we are interested in the intermediate level, where actual cognitive processes happen, which are inaccessible to the consciousness.
These processes are not only factually special principle unconsciously. Typical representatives: Chomsky, Marr, Fodor.
I 256
Explanation/SearleVsCognitivism: Thesis: many of our cognitive science explanations do not have the explanatory power, we attach to them. To save them, we will have to make a reversal of its logical structure: as it took place during the transition from pre-Darwin biology to the biology à la Darwin.
I 256/257
The brain produces states of consciousness, and that is all. As for the mind, this is already the whole story. There are the blind neurophysiological processes and there is consciousness, otherwise however there is nothing. No rule-following, no mental information processing, no unconscious inferences, no mental models, no original drafts, no two and a half dimensional images, no language of the mind, and no universal grammar.

Searle IX
John R. Searle
"Animal Minds", in: Midwest Studies in Philosophy 19 (1994) pp. 206-219
In
Der Geist der Tiere, D Perler/M. Wild Frankfurt/M. 2005
Descartes, R. Kant Vs Descartes, R. Danto I 179
KantVsDescartes: The cogito, that "I think" is not an indubitable proposition, but something that accompanies every sentence you claim. KantVsDescartes: cogito does not penetrate, but accompanied thinking.

Kant I 73
Existence/cogito/Kant: feeling of existence has no concept. Nothing can be proven here. VsDescartes: "I ​​think, therefore I am," error: to infer from the concept to the existence of a thing.

Field I 80
KantVsOntological proof of God’s existence/KantVsDescartes: (KdrV, B622,3 4): You can never assert the categorical (non-conditional) existence of something. Justification: Contradictions usually originate from the fact that one or more Objects are postulated, and then assumptions that are mutually inconsistent: e.g. a triangle and it being quadrilateral.
But there is no contradiction to deny the existence of a triangle!
For we have not made any conflicting assumptions. ((s) by only assuming a triangle.)
Kant: The same applies to the notion of an "absolutely necessary being": if we deny its existence, we deny it with all its predicates, but then no contradiction can arise.
Nothing can be negated with all predicates and yet leave a contradiction. (s) So there is no necessary existence.
Field: it can not be contradictory to deny the existence of numbers, because they have no mysterious force to leave a contradiction if they are not there. (s) Has the triangularity a mysterious force if there is no triangle? No, but that is a predicate without a carrier and not comparable here).

Stegmüller IV 362
Proof of God’s existence / Kant Descartes: Four points (CPR A 594 p): 1. "If I pick up the predicate in an identical judgment and keep the subject, the result is a contradiction." I lift both together, there is no contradiction. E.g. I cannot lift the omnipotence if God is the same as omnipotence. But if I say God is not, neither omnipotence nor any other of his predicates are given. IV 363 StegmüllerVsKant: One can ask why Kant is so sure that no negative existential proposition is self-contradictory and why therefore no existence statement of the form "there is an x" can be an analysis.
2. Kant (A 597): "You have already committed a contradiction when you brought into the concept of a thing, of which you only wanted to think its possibility,the notion of its existence". MackieVsKant: This is unfair! Kant’s argument is based on the idea that Descartes has an "an open mind" concerning the existence of God or not, hence something is read into the concept of existence. But Descartes does not pretend that he is open-minded regarding the response, he is rather completely sure regarding the existence. But then he does not postulate what needs to be proofed as proofed.
3. Kant (A 598): Analytic/Synthetic distinction: there can be no analytical statements about existence. (However, he does not justify this claim).
IV 364
VsKant: Analytical judgments on existence are in arithmetic, e.g. there is a prime number between 10 to 20 Frege: All arithmetic truths are analytic.
4. Kant: The logic of existence statements reflect an incorrect grammar: the auxiliary verb "be" is ambiguous here when it is used as a means of predication and existence. (> Copula).
MackieVsKant: Kant stops halfway: If to "exist" is not a predicate, then what is it?
Existential quantifier: exists only since Frege.
IV 365
MackieVsDescartes: That is a deadlier argument: the existential quantifier cannot be an attribute and cannot express perfection, which may possess a thing or not. E.g. therefore the Revenus resident cannot be refuted, which has no necessary perfection but only an artificial perfection. There is no distinction between natural and artificial perfection in the existential quantifier, there is now no distinction between natural and artificial perfection. Then Descartes’ argument about the distinction of natural/artificial, with God the only exception of a being no longer with natural perfection, is not valid anymore. DescartesVsFrege: his only rebuttal would be if he could prove that a "this tree" or "I" or "God exists" ((s) so (ix) Fx (iota operator, indicator statement) exists MackieVsDescartes / Stegmüller.: In any case, he has not done this.


Strawson V 22
"Refutation of idealism"/ Kant Descartes: So that self-consciousness is possible, it must be at least possible to distinguish between consequences of our experiences on the one hand, and consequences of the objects of our experience which they show independently. For that, the items must be so designed that they exist in a stable framework. The necessary differences of temporal relations must be taken within the experience. We must therefore have a direct and non-deductive awareness of objects in space. "The consciousness of my own existence is at the same time the non-deductive consciousness of the existence of other things beside me." Terms / Kant: not any amount of terms is sufficient for us, there must be concepts of persistent and re-identified Objects among them.
V 23
StrawsonVsKant: In the analogies, he always tries to squeeze more out of the arguments than there actually is. Self-awareness/Consciousness/Kant/Strawson: The distinctions must be created in the concepts themselves, because there is no such thing as a pure perception of the reference system!
V 103
KantVsDescartes: self-awareness is only possible through the perception of external objects. Substance, cause and community (or reciprocal interaction is a necessary condition for objective experience. And these concepts become only meaningful regarding external objects. Strawson: Kant relies here very little on his theories from the transcendental aesthetic as premises for its arguments in the analysis.

Strawson V 140
Def Soul/Descartes/Strawson: All of us know by the mere fact of conscious awareness that he exists as a (Cartesian), thinking substance, e.g. that it is capable as an intangible, lasting, not composite individual subject of ideas and experiences as well as an existence in complete independence of a body or of matter. KantVsDescartes: Which infringes the principle of sense: there is no empirical application criteria for this claim.
KantVsDescartes, KantVs rational psychology: Analysis of the origins of appearance: Mix-up of the unity of experiences and the experience of unity.
V 143
KantVsDescartes: After all, it is the unity of consciousness, which we, if the semblance has us under control, take erroneously for awareness of a unified subject.
V 145
Def rational psychology/(Descartes): Asserts that every person has immediate safety regarding the existence of his soul as an immaterial substance. KantVsDescartes: However,the only criteria for it would be "the same man, the same soul". Deathblow for rational psychology.
I. Kant
I Günter Schulte Kant Einführung (Campus) Frankfurt 1994
Externe Quellen. ZEIT-Artikel 11/02 (Ludger Heidbrink über Rawls)
Volker Gerhard "Die Frucht der Freiheit" Plädoyer für die Stammzellforschung ZEIT 27.11.03

Danto I
A. C. Danto
Connections to the World - The Basic Concepts of Philosophy, New York 1989
German Edition:
Wege zur Welt München 1999

Danto III
Arthur C. Danto
Nietzsche as Philosopher: An Original Study, New York 1965
German Edition:
Nietzsche als Philosoph München 1998

Danto VII
A. C. Danto
The Philosophical Disenfranchisement of Art (Columbia Classics in Philosophy) New York 2005

Field I
H. Field
Realism, Mathematics and Modality Oxford New York 1989

Field II
H. Field
Truth and the Absence of Fact Oxford New York 2001

Field III
H. Field
Science without numbers Princeton New Jersey 1980

Field IV
Hartry Field
"Realism and Relativism", The Journal of Philosophy, 76 (1982), pp. 553-67
In
Theories of Truth, Paul Horwich Aldershot 1994

Carnap V
W. Stegmüller
Rudolf Carnap und der Wiener Kreis
In
Hauptströmungen der Gegenwartsphilosophie Bd I, München 1987

St I
W. Stegmüller
Hauptströmungen der Gegenwartsphilosophie Bd I Stuttgart 1989

St II
W. Stegmüller
Hauptströmungen der Gegenwartsphilosophie Bd 2 Stuttgart 1987

St III
W. Stegmüller
Hauptströmungen der Gegenwartsphilosophie Bd 3 Stuttgart 1987

St IV
W. Stegmüller
Hauptströmungen der Gegenwartsphilosophie Bd 4 Stuttgart 1989

Strawson I
Peter F. Strawson
Individuals: An Essay in Descriptive Metaphysics. London 1959
German Edition:
Einzelding und logisches Subjekt Stuttgart 1972

Strawson VII
Peter F Strawson
"On Referring", in: Mind 59 (1950)
In
Eigennamen, Ursula Wolf Frankfurt/M. 1993
Descartes, R. Locke Vs Descartes, R. I 27
Innate ideas/LockeVsScholastics/LockeVsDescartes: there are no innate ideas! Neither in speculative nor in practical (moral, theological) thinking, not even in the form of "maxims", i.e. immediately plausible principles. 1. Speculative principles: if they were innate, they would have to be demonstrable in people not yet spoiled by prejudices, as, for example, in children or mentally weak people, and they are not!
2. If truths were innate in the form of sentences, then these would also have to be the associated terms, even the conclusions from these sentences! Such assumptions, however, extend the range of innate concepts and sentences into the impossible.
3. Maxims: the spontaneous consent to them means that they were not known before! But innate must always be present.
ChomskyVsLocke/(s): would Object that Grammar rules also come into consciousness first. This is about the ease of learning).
Innate ideas/Curls: the assumption that thinking begins with the application of innate laws of thought or first principles that are more than mere instrumental thinking is a deception.
I 45
Body/Stretch/res extensa/LockeVsDescartes: stretch and body are therefore not identical! It is also not at all clear that the mind must let them be distinguished from the body. (Risked the dangerous accusation of materialism). The idea of expansion and the idea of the body are different.
Expansion: does not include strength or resistance to movement (>inertia).
Space: cannot be divided, otherwise surfaces would come up!
VsCartesians: they have to admit that they either think of bodies as infinite in view of the infinity of space, or they have to admit that space cannot be identified with bodies.
I 52
Res cogitans/LockeVsDescartes: Descartes: to strictly separate the world of bodies from the world of thought.
Locke: mentions to consider whether there could not be extended things, thus bodies that think, something flowing matter particles. In any case, it cannot be ruled out that God in his omnipotence "matter systems" may have
I 53
given or "overturned" the power of perception and thought. Contemporary theologies felt provoked by this, especially his Kontrahend Stillingfleet.
LockeVsDescartes: also leads to problems with human identity (see below).
I 54
Identity/LockeVsDescartes: Problem: the relationship between substance and person when the ability to think is attributed solely to an immaterial substance. For example, it would be conceivable that someone could be convinced that he was the same person as Nestor. If one now presupposes the correctness of the Cartesian thesis,
I 55
it is conceivable that a contemporary human being is actually the person Nestor. But he is not the human being Nestor, precisely because the idea of the human cannot be detached from his physical form.
That is abstruse for us today. (> Person/Geach).
Locke relativizes the thesis by saying that it is not the nature of the substance that matters to consciousness, which is why he wants to leave this question open - he conveys the impression that he is inclined towards the materialistic point of view.
II 189
Clarity/LockeVsDesacrtes: no truth criterion, but further meaning: also in the area of merely probable knowledge.
II 190
Clarity/LockeVsLeibniz/LockeVsDescartes: linked to its namability. Assumes the possibility of a unique designation. (>Language/Locke).
II 195
Knowledge/Locke: according to Locke, intuitive and demonstrative knowledge form a complete disjunction of possible certain knowledge. VsDescartes: this does not consist in a recognition of given conceptual contents, which takes place in their perception, but constitutes itself only on the empirical basis of simple ideas in the activity of understanding.

Loc III
J. Locke
An Essay Concerning Human Understanding
Evans, G. Peacocke Vs Evans, G. I 169/170
Demonstratives/Evans: perceptually demonstrative ways of givenness are possible, because these conditions are fulfilled: in a normal perception situation, there is an information link between subject and Object, and also the subject knows or is able to find out where the Object is.
If the subject has the general ability to know what propositions makes of the form
"π = p" true for any π (where π is an identification of a public place without index words (in a non-indexical frame of reference)) if p is the notion of ​​a place in its egocentric space. If it is also able to locate the Object in its egocentric space, we can say that it has an idea of the Object.
Idea/Notion/Evans/Terminology/Intension/Way of Givenness/Peacocke: Evans "Idea" (notion) corresponds to my way of givenness "mode of presentation".
Idea/Evans: Thesis: we can conceive the idea of an Object a as consisting in its knowledge of what it is to be true for an arbitrary sentence of the form "δ = a".
Peacocke: where "δ" is the area of ​​the basic ideas of an Object.
Fundamental Idea/Evans: is what you have if you think of an Object as the possessor of the fundamental ground of difference that it actually has.
Peacocke: i.e. what distinguishes an Object from all others.
I.e. for material Objects type and location.
PeacockeVsEvans: we have already seen cases where the thinker was unable to locate the Object in his egocentric space: E.g. the craters on the moon.
I 171
E.g. apple in the mirror cabinet. But it still seems possible to think about it, for example, wonder where it is!
It is true that it is possible to at least provide a rough direction in egocentric space, but that is hardly sufficient for the knowledge condition of Evans.
In the case of the memory image, it is clearer that no localization in the current egocentric space is needed.
pro Evans: there must be additional imaginable evidence, e.g. experience or tools for localization (if necessary, even space travel!).
If that were not imaginable, we would have to assume that the subject was not able to think of the Object in public space!
pro Evans: an information link is not sufficient to think demonstratively about the Object.
VsEvans: but that is less than to demand that the thinker can locate the Object at present.
Weaker Requirement: Instead, a general ability of the subject can locate the Object, if necessary, is sufficient.
Evans: if you cannot locate an Object, you can still think of it in the mixed demonstrative descriptive way of givenness: "that which causes my experience".
But in normal cases this is a wrong description!
Peacocke: it also seems to be wrong in the examples of the lunar craters, the apple in the mirror cabinet.
PeacockeVsEvans: trange asymmetry:
Idea/Evans: an idea a of ​​a place in a self-centered space is an adequate idea of ​​a place in the public space.
Holistic/Evans: if an arbitrarily fundamental identification of a location is possible, it is holistic. (Varieties of reference, p. 162).
Peacocke: this knowledge is grounded in a general ability to put a cognitive map of the Objective spatial world over our own egocentric space.
I 172
E.g. in some cases this will not be possible, for example, when you are kidnapped, or ended up in an unknown area, etc. Point: even in such cases, you can still use the demonstrative pronoun "here" (in reference to Objects). I.e. the thoughts are still thoughts about public space! ((s) and the self-centered space).
Idea/Demonstrative Way of Givenness/PeacockeVsEvans: so his theory does not demand any ability to give a public, non-egocentric individuation our thoughts to have thoughts about a place in the public space at all.
Analogy/Peacocke: exactly analogous Objections can be made in the case of demonstrative ways of givenness: E.g. Suppose a subject perceives an Object of type F in the manner H.
Then F is the token way of givenness.
Then we can introduce: [W, Fs] for the perceptual "this F".
Then there is exactly one proposition of the form "p = localization of [W, Fs] now", which is true, and the subject knows what it is for it that it is true for it.
PeacockeVsEvans: why should we demand here, but not in the earlier example, that the subject also knows which p (or which  in the earlier case) is mentioned in this one true proposition?
This is particularly absurd in the case of the lost subject.
PeacockeVsEvans: his theory allows that [W, Fs] is an adequate idea here, although the subject has no fundamental idea of the Object.
Peacocke: but if we insisted that it could have a fundamental idea if he had more evidence, then why is an analogous possibility not also sufficient for adequacy in terms of the egocentric space?
I 173
There seem to be only two uniform positions: 1) Identification/Localization/Idea/Demonstratives/Liberal Position: sufficient for a genuine way of givenness or adequate ideas are the general ability of localization plus uniqueness of the current localization in the relevant space.
2) Strict position: this is neither sufficient for genuine ways of givenness nor for adequate ideas.
PeacockeVs: this can hardly be represented as a unified theory: it means that, if you are lost, you cannot think about the Objects that you see around you. That would also mean to preclude a priori that you as a kidnapped person can ask the question "Which city is this?".
Demonstratives/Peacocke: Thesis: I represent the uniformly liberal position
Demonstratives/Evans: Thesis: is liberal in terms of public space and strictly in terms of egocentric space!
ad 1): does not deny the importance of fundamental ideas. If a subject is neither able to locate an Object in the public nor in egocentric space ((s) E.g. he wakes up from anesthesia and hears a monaural sound), then it must still believe that this Object has a fundamental identification. Otherwise it would have to assume that there is no Object there.
Anscombe: E.g. a subject sees two matchboxes through two holes which (are manipulated) so arranged that it sees only one box, then the subject does not know what it means for the sentence "this matchbox is F" to be true.
The uniformly liberal view allows the subject to use demonstratives which depend on mental images, even if it has no idea where in the public space and when it has encountered the Object.
EvansVs: representatives of this position will say that the knowledge of the subject is at least partial,
I 174
because this idea causally results from an encounter with the object. But that makes their position worse instead of better: for it completely twists the grammar and logic of the concept of knowing what it is for the subject that p is true. Ability/PeacockeVsEvans: but a capability can also consist in the experience of finding out the right causal chains in a given environment: the same goes for the localization of an Object point seen in the mirror in egocentric space.
PeacockeVsEvans: his distinction seems unreal: it may be simultaneously true that someone has a relation R to the Object due to causal relations, and be true that the possibility of being in this relation R is a question of the abilities of the subject.
E.g. (Evans) to recognize the ball:
Peacocke: this is not a sensory motor skill, but rather the ability to draw certain conclusions, which however require an earlier encounter.
This also applies to e.g. the cognitive map, which is placed over the egocentric space:
PeacockeVsEvans: in both cases it does not follow that the presented Object, remembered or perceived, is thought of explicitly in causal terms: the way of givenness is truly demonstrative.
   
First Person/PeacockeVsEvans: the second major Objection concerns thoughts of the first person: the different examples of immunity to misidentification, which contain the first person, roughly break down into two groups:
a) here, immunity seems absolute: E.g. "I am in pain".
I 175
b) Here, the immunity seems to depend on presuppositions about the world: if these assumptions are wrong, they open the possibility of picking out something wrong without stopping to use the word "I". These include: E.g. "I was on the ocean liner": memory image.
E.g. "I sit at the desk": visual, kinesthetic, tactile perceptions.
The distinction between a) and b) may be made by the constitutive role:
"The person with these conscious states."
Infallibility/Tradition/Evans: (absolutely immune judgments): the judgment to be a judgment of a specific content can be constituted by the fact that this judgement responds to this state.
Peacocke pro.
PeacockeVsEvans: Problem: can this infallibility be connected to the rest of Evans' theory? Because:
I/Evans: Thesis: the reference of "I" may fail!
Peacocke: how is that compatible with the absolute immunity of "I am in pain"?
Conditionalisation: does not help: E.g. "if I exist, I am in pain" that cannot fulfill the purpose: the existence of the idea still needs the reference of "I".
Similarly: E.g. "If my use of "I" refers, I am in pain":
because "my use" must be explained in terms of the first person.
Question: Can we use memory demonstratives which refer to previous use of first-person ways of givenness?
E.g. "If those earlier uses of "I" speak, I am in pain." (Point: not "my uses").
PeacockeVs: that does not help: Descartes' evil demon could have suggested you the memories of someone else. (>Shoemaker: q-memories.)
I 176
Constitutive Role/Brains in the Vat/BIV/EvansVsPeacocke: the constitutive role of [self] would not explain why the brains in the vat would be able to speak in a demonstrative way about their own experiences: Mental States/Evans: differ from all other states and Objects in that they refer demonstratively to their owners.
Pain is identified as an element of the Objective order.
Then someone can have no adequate idea of ​​these mental states if he does not know to which person they happen.
Peacocke: we can even concede thoughts about its pain to the brain in a vat, provided that it can give a fundamental identification of the person who has the pain.
Peacocke: No, the nerves must be wired correctly. I.e. this is not true for the brains in the vat. So we can stick to the liberal point of view and at the constitutive role and the idea of a person.
Also to the fact that the mental states are individuated on the person who has them.
Individuation/Mental States/PeacockeVsEvans: not through localization (like with material Objects), but through the person.
I 177
E.g. Split-Brain Patient/Peacocke: here we can speak of different, but qualitatively equivalent experiences. From this could follow two centers of consciousness in a single brain. But: after the surgery we should not say that one of the two was the original and the other one was added later.
E.g. olfactory sensation of the left and right nostril separate. Then there are actually separate causes for both experiences. ((s), but the same source.)
Peacocke: it does not follow that in normal brains two consciousnesses work in harmony. Here, the sense of smell is caused by simultaneous input through both nostrils and is thus overdetermined.

Peacocke I
Chr. R. Peacocke
Sense and Content Oxford 1983

Peacocke II
Christopher Peacocke
"Truth Definitions and Actual Languges"
In
Truth and Meaning, G. Evans/J. McDowell Oxford 1976
Geach, P. Quine Vs Geach, P. V 18
Perception/Quine: has more to do with consciousness than with the reception of stimuli. But it is also accessible to behavior criteria. It shows in the conditioning of reactions.
V 18/19
Dispositions/Quine: habits resulting from conditioning.
V 89
Identity/Geach: (Reference and generality, p 39f.): Only makes sense with reference to a general term like e.g. "the same dog". QuineVsGeach: this is certainly true for the beginning of language learning.
Identity/Pointing/Quine: Problem: there is no point in pointing twice and saying, "This is the same as that." Then you could still ask "The same what?".
E.g. you could have been pointing once to the dog and merely to the ear the next time.
Solution: you can easily say that a is identical with b. Whether a is the same dog or the same ear depends on whether a is a dog or an ear.
QuineVsGeach: this makes his relativism untenable once you get accustomed to the identity way of speech.
Identity/Quine: in a deeper sense still relative. (see below § 30)

V 129
Pronouns/Pronouns/Quine: are the archetype of variables in logic and mathematics. Everyday Language: here pronouns are an important part of relative clauses.
Relative Clause/Language Learning/Quine: E.g. "I bought Fido from a man who had found him."
Function: the relative clause makes it possible to separate the Object of what the sentence says about it.
Relative Clause: becomes a general term if the pronoun for the name of the Object is out in front: E.g. "which I bought from a man who had found him" is a general term!
This general term says the same thing of Fido as the original sentence.
Relative Clause/GeachVsQuine: (Reference and generality, p.115 122, also "Quines syntaktische Einsichten").
Relative Pronoun/Geach: instead, conceive it as meaning "and he": e.g. "I bought Fido from a man and he had found him." ((s) paratactic analysis).
Or with "when he" or "since he".
V 130
Geach calls this the "Latin prose theory". Def Latin Prose Theory/Geach: Thesis: it's wrong to consider "who had found him" as a terminus or independent grammatical entity at all.
Donkey Sentence/Geach's Donkey/Quine: E.g.
Everyone who owns a donkey beats it;
Some donkey owners do not beat them.
Problem: that would turn into nonsense:
Every donkey owner beats it
Some donkey owners do not beat it.
Solution/Geach: analysis of the relative pronoun "who" with "if he":
Every person, if he has a donkey, beats it.
Example (by Emmon Bach): ((s)> Brandom, Bach Peter's sentences)
A boy who fooled her kissed a girl that loved him.
Geach: here, you cannot consider "boy who fooled her" as a separate term, because then the floating pronoun "her" would have no reference, not even to "girl who loved him", because the floating pronoun "him" would then have no reference.
Solution/Geach:
A boy kissed a girl and she really loved him, but he only fooled her.
Quine: pro Geach.
((s) sequence of main clauses.)
V 131
Relative Clause/Bach Peter's Sentences/Donkey Sentence/Geach's Donkey/Geach/Quine: Geach focuses on the quantification (1) (Ex) (x is a man and I bought Fido of x and x had found Fido)
(2) (x) (y) (if x is a man and y is a donkey and x has y, then x beats y).
(3) (Ex) (Ey) (x is a man, and y is a a donkey and x has y and not (x beats y))
(4) (Ex) (Ey) (x was a boy and y was a girl and y kissed y and y really loved x, but x merely fooled y).
QuineVsGeach: the description of the correct Grammar is one thing, a plausible description of a child's language learning is another. It would be nice if both matched, which is to be expected according to Hall, Bloomfield and Chomsky.
QuineVsGeach: before this is proven, I tend to a more dualistic view. Geach's Latin prose theory correctly describes the Grammar, but not the learning process. Most examples of relative clauses correspond to the Fido example.
The child is torn between analogies,
V 132
which are in the end described properly by Geach. Relative Clause/Quantification/Language Learning/QuineVsGeach: a reformulation of the relative pronoun depending on circumstances in "and he" or "if", etc. is too complicated. In addition, the quantification would need to be learned before the relative clauses. Instead, the child comes to the quantification the other way round, through the relative clause.

Strawson I 198
QuineVsGeach/QuineVsFrege: Singular terms can take the places of quantifiable variables, general expressions cannot. Singular Term: quantifiable, General Term: not quantifiable.

Quine I
W.V.O. Quine
Word and Object, Cambridge/MA 1960
German Edition:
Wort und Gegenstand Stuttgart 1980

Quine XIII
Willard Van Orman Quine
Quiddities Cambridge/London 1987

Strawson I
Peter F. Strawson
Individuals: An Essay in Descriptive Metaphysics. London 1959
German Edition:
Einzelding und logisches Subjekt Stuttgart 1972

Strawson VII
Peter F Strawson
"On Referring", in: Mind 59 (1950)
In
Eigennamen, Ursula Wolf Frankfurt/M. 1993
Goodman, N. Chomsky Vs Goodman, N. I 287
Language learning/language acquisition/Goodman: Second language is not problematic because the acquisition of the first language is the acquisition of a "secondary symbolic system". ChomskyVsGoodman: that could have some weight if it could be shown. (For example, for the distinction of surface structure and depth structure).
But we have no empirical evidence.
---
I 288
ChomskyVsGoodman: Acquisition of first and second language: Fallacy: If we learn the second language easier by means of explanations from the first language, we would have had to acquire a language before the first language in order to acquire the first language (which is particularly easy). (Regress). Goodman: Acquisition of the first language is acquisition of a "secondary symbolic system" and therefore corresponds to the acquisition of the second language.
Chomsky's: the primary symbolic systems that he has in mind are rudimentary and cannot be used in the same way as a first language in acquiring the second language.
GoodmanVsChomsky: his theses cannot be checked because we do not have examples of "bad languages".
---
I 289
ChomskyVsGoodman: There are dozens of books in which features of a universal grammar are formulated and their empirical consequences are examined, whereby each such property specifies "bad" languages. ---
I 290
Grue/ChomskyVsGoodman: affects more of a border problem. The initial question is too vague. You can easily find a property, even a fairly general one, of the language "grue bleen", which is not the property of a "language like German".
E.g. Chomsky: the predicate "be equal" (Structure of Appearance) applies only to Objects instead of to Qualia.
Now the language grue bleen has the peculiar property: "If an Object A before t and an Object B after t are examined, and if both are determined to be grue (or bleen), then we know that they are not like each other.
But there is no such t that we could predict of these Objects that they will not be equal. They could just as well be equal if both are grue (or bleen).
Chomsky: it is undoubtedly a general property of natural languages that they behave more like German than "gruebleen".
Thus, there is no difficulty in establishing a distinction between such languages as grue bleen and such as German.
This would not suffice Goodman, of course, because one could still construct more refined examples.
As long as it is only about vague terms like "like German" or "like Gruebleen", Goodman's requirement is impossible to fulfill.
---
I 291
ChomskyVsGoodman: It may be relevant to induction, but not to linguistics, just as little as for any other science, such for the question of why embryos get arms and no wings within a given framework of conditions. ((s) is irrelevant because once conceptual, once empirical.)
Chomsky: with this we cannot explain at all why the learner does not acquire grue as a generalization basis. Undoubtedly this follows from certain properties of the sensory system.
Congenital ideas/ChomskyVsGoodman: it does not seem incomprehensible to me that any aspect of the "final state" of an organism or automaton is also an aspect of its "initial state". And this before any interaction with his environment!
---
I 292
Innate ideas/ChomskyVsGoodman: in his essay, Goodman at least once admits that the mind contains ideas in some sense. Then it is obviously not incomprehensible that some of these ideas are "implanted as an original equipment" to the mind.

Chomsky I
Noam Chomsky
"Linguistics and Philosophy", in: Language and Philosophy, (Ed) Sidney Hook New York 1969 pp. 51-94
In
Linguistik und Philosophie, G. Grewendorf/G. Meggle Frankfurt/M. 1974/1995

Chomsky V
N. Chomsky
Language and Mind Cambridge 2006
Harman, G. Putnam Vs Harman, G. Harman II 421
Truth/HarmanVsPutnam: it is not merely idealized rational acceptability. It involves a relationship between a remark or a thought and the way how things are in the world.
Putnam/Harman: is right when he equates the decisive point with a determination to the localization of all the facts in a world.
Harman: when I suppose, thesis, there is one clear causal physical order, I ask myself the following questions: "What is the place of the mind in the physical world?", "What is the place of values in the world of facts?" I believe that it is a serious philosophical error, if we believe we can avoid these issues.
PutnamVsHarman: a position as Harman's leads to two implausible conclusions:
1. Identity thesis of body and mind. (HarmanVs! I do not think that it follows from the assumption of a single causal order, rather to functionalism, that Putnam himself represented)
2. moral relativism. (Harman pro! There is nothing problematic).
Harmans II 428
Truth/HarmanVsPutnam: I do not think that he would consider it as a good argument for the conclusion that truth is the same as >consistency: Problem: but then his argument does not show that truth is an idealization of rational acceptability.
Harman II 434
Competence/Chomsky/Putnam: (Chomsky Syntactic Structures) promised us that there would be a normal form for grammars and a mathematical simplicity function that would explain everything precisely. Here you would have to look at various descriptions of the speaker's competence, which are given in the normal form, and measure the simplicity of every description, (with the mathematical function) in order to find the easiest. This would be "the" description of the speaker's competence. Putnam: actually Chomsky owes us also a mathematical function with which one measures the "goodness", with which the competence description fits with the actual performance.
Chomsky/Putnam: the idea of ​​mathematization has since been abandoned. The idea currently rests that the speaker's competence could be given by an idealization of the actual speaker's behavior, on an intuitive notion of a "best idealization" or "best explanation".
Justification/PutnamVsChomskyPutnamVsHarman: to assume that the concept of justification could be made physicalistically through identification with what people should say in accordance with the description of their competence, is absurd.
Harman II 435
Harman/Putnam: but would say that there is a difference whether one asks if the earth might have emerged only a few thousand years ago,
Harman II 436
or whether one asks something moral, because there are no physical facts, which decide about it. PutnamVsHarman: if the metaphysical realism with Harman (and with Mackie) has to break, then the whole justification of the distinction facts/values is damaged.
Interpretation/explanation/Putnam: our ideas of interpretation, explanation, etc. come from human needs as deep as ethical values.
Putnam: then a critic might say of me, (even if he remains metaphysical realism): "All right, then explanation, interpretation and ethics are in the same boat" ("Companions in Guilt" argument).
Putnam: and this is where I wanted it to be. That was my main concern in "truth, reason and history." (Putnam thesis explanation, interpretation and ethics are not in the same boat" ("companions in guilt" argument: in case of partial relativism the total relativism is near. PutnamVsHarman).
Relativism/Putnam: There is no rational reason to support ethical relativism, but not at the same total relativism.
Reference/Harman/Putnam: Harman's answer is that the world has a unique causal order.
Harman II 437
PutnamVsHarman: but that does not help: if my linguistic competence is caused by E1, E2 ... , then it's true that it was caused* by E*1, E*2 ... whereby* the corresponding entity designates in a non-standard model. ((s)>Löwenheim) Problem: why is reference then determined by cause and not by cause*?
Reference/Physicalism/Putnam: the only answer he could give, would be: "because it is the nature of reference". This would mean that nature itself picks out Objects and places them in correspondence to our words.
David Lewis/Putnam: has suggested something similar: ... + ...

Putnam I
Hilary Putnam
Von einem Realistischen Standpunkt
In
Von einem realistischen Standpunkt, Vincent C. Müller Frankfurt 1993

SocPut I
Robert D. Putnam
Bowling Alone: The Collapse and Revival of American Community New York 2000

Harman I
G. Harman
Moral Relativism and Moral Objectivity 1995

Harman II
Gilbert Harman
"Metaphysical Realism and Moral Relativism: Reflections on Hilary Putnam’s Reason, Truth and History" The Journal of Philosophy, 79 (1982) pp. 568-75
In
Theories of Truth, Paul Horwich Aldershot 1994
Heidegger, M. Quine Vs Heidegger, M. V 127
Identity/Everyday Language/Individuation/Reference/Quine: also identity is part of our referential apparatus, but it is obscure in everyday language, because we use it without clear individuation principle. E.g. Do two editions of a novel have the same hero? How unlike may the heroes be? Or e.g. how unlike may the editions be to still be considered as versions of the same novel?
E.g. Was Baal the devil? E.g. Did the Indians rever God by worshiping the Great Spirit?
Identity/Possible Worlds/PoWo/Quine: all these examples fall under the issue of cross-world identity. Identity in various possible worlds.
Differently:
Attributes/Identity/Quine: E.g. when attributes are coextensive, they are not necessarily the same attribute. But when are they anyway?
Wrong solution: some say in case of "necessary co-extensivity" the two attributes are identical.
QuineVs: that only shifts the problem.
Ontology/QuineVsHeidegger: we do not clarify ontological ambiguities by taking everyday language literally and sifting through it. (>Existence, >value of a bound variable).
((s) primacy of language not in ontology).
V 128
Solution/Quine: it is the other way round: one comes up with something and gears language towards it! Existence/Ontology/Language Learning/Quine: the existing things are genetically nothing but an interplay of grammatical analogies that cover up the differences in the forms of learning. In the center is talk of Objects. Ontology begins with the generalization of Object study. (see above: e.g. color words, which, as you learn, do indeed not refer to individual things).
Grammar is thus simplified, ontology is multiplied.

Quine I
W.V.O. Quine
Word and Object, Cambridge/MA 1960
German Edition:
Wort und Gegenstand Stuttgart 1980

Quine XIII
Willard Van Orman Quine
Quiddities Cambridge/London 1987
Heraclitus Quine Vs Heraclitus I 296 ff
Everyday language has the annoying habit of grammatically highlighting time relations at the expense of relations relations of weight or color. In the canonical notation you usually drop the temporal distinctions. Even in mathematics: we feel the "is" differently after "seven" than after "Maria".
Re-forming. E.g. "I called him, but he is asleep" becomes: "I call him then, but he sleeps at that time".
E.g. "Earlier than now George marries Maria and now Maria is a widow, therefore Georg earlier marries someone who is now a widow."
I 298
QuineVsHeraclitus: It's not a bigger problem to step twice into the same river than it is to do so twice at two different locations. (Goes back to the different weighting in our grammar).
III 270
Identity/Time/Change/Transformation/Heraclitus/QuineVsHeraclitus: how can you say that a thing that changes its substance does not remain identical with itself? The key is not in the concept of identity, but in the concepts "Object" and "time".
Def Object/Object/Thing/Quine: in every moment the sum of the simultaneous current states of atoms ​​distributed in the area or other small physical particles.
And over time it is the sum of its successive current states.
QuineVsHeraclitus: we can step into the same river twice. What we cannot do is step twice into the same temporal stage (time stage) of the river. (At least not if this part is shorter than the time we need to climb into it ((s)(twice)).
III 271
((s) Transformation/Change/Quine/(s): depends on the choice of the time periods under comparison.) Equal Sign/Quine: "=" is an ordinary relative term (rel term).
The equal sign is necessary, because two variables may relate to the same or to different Objects.
From a logical point of view, the use of the equal sign between variables is fundamental, not between singular terms.

V 186
Ontology/QuineVsHeraclitus: we forced his talk of time and river into a clear structure of general term and singular term and the reference to objects. Thus we have a simpler ontology.
VII (d) 65
Identity/Heraclitus/Quine: E.g. you cannot bathe in the same river twice. Solution: you can, but not twice in the same "river stages".
A river is a process in time. Unlike its stages.
Water: to be a multiplicity of water molecules.
VII (d) 66
River Stage: is simultaneously a water stage. But two stages of the same river are not always stages of the same water. ((s) division into two types of stages to explain the change). Quine: in our fast-paced world you could bathe twice in the same water but in different rivers!
A: current stage of the river Cayster in Lydia
b: stage of the Cayster two days later
c: Current (two days later) state of the water molecules from river stage a.
Half of them is further downstream, the other half in the Aegean Sea.
a and b: are in "river relation".
a and c: are in "water relation".
River: as an entity is thus introduced as a single thing, namely as a process or time-consuming Object that you say identity instead of "river relation".
Identity: but you cannot say that a and b are the identical, they are merely river-related. But if we point to a and after two days to b, then we should express that we do not point to stages, but to the same river, which contains both. The assumption of identity is essential.

Quine I
W.V.O. Quine
Word and Object, Cambridge/MA 1960
German Edition:
Wort und Gegenstand Stuttgart 1980
Intention Based Semantics Schiffer Vs Intention Based Semantics I 258
SchifferVsIntention-based semantics/SchifferVsIBS: much worse: from normal speakers too much knowledge is required. For example, that he knows the function that maps sentences to propositions. Solution/Lewis: (Lewis 1975):
Actual speech ratio / population / Lewis (Lewis 1975): L is a language in G only if it's common knowledge in G that members of G "never attempt to express a proposition of L, which is not true in L "(p 167). Then Lewis would respond to the above Objections:
I 259
Lewis: the normal human being does not need a term of L to expect that his fellows are truthful. He just needs proper expectations about how they should behave. He expects them to act in accordance with a regularity of truthfulness. But we would - and not he - describe this as regularity. He might have an internally represented Grammar, and being able to have the potentially infinite number of expectations, but this is not critical. (p. 180f).
Schiffer: Problem: it is not entirely clear how this is to avert the above Objection: to know that a fellow human being will never say a false sentence, a member of the population must know the function. And in addition he needs a manner of givenness (givenness, "concept"). And that is too much for the knowledge that can be attributed to normal people.
Lewis: seems to want to attribute the following knowledge:
For all s, p, if L(s) = p, then it is common knowledge, in G, that members of G would not express s, if p is not true.
Schiffer: I do not know whether that's adequate for Lewis, it does not help the IBS: the idea is to redraft IBS definitions in a way so that all references to L are outside of that-propositions. ((s) so that the speaker does not affect the language itself.).
Pointe: then the individual speakers must know only sentences and individual propositions.

Schi I
St. Schiffer
Remnants of Meaning Cambridge 1987
Kripke, S. A. Anscombe Vs Kripke, S. A. Frank I 84
I/Descartes: not a kind of body. I could assume that I don’t have a body. I/Augustine: "the mind knows of itself, that it is thinking." "It knows its own substance."
Kripke/Anscombe: K. tried to rehabilitate Descartes’ argument for his dualism.
AnscombeVsKripke: he neglects his first person character by making it an argument for the non-identity of Descartes with his own body.
I 85
According to this, Descartes would have had to doubt the existence of Descartes as a human being, and in any case the existence of this figure in the world of his time, of this Frenchman, christened René... Descartes/AnscombeVsKripke: "I am not Descartes" was for him like "I’m not a body!" Forcing the argument into the third person perspective by replacing "I" with "Descartes" means to neglect this.
Descartes never thought, "Descartes is not Descartes" (which according to Anscombe is ascribed to him by Kripke).
I 85/86
AnscombeVsKripke: this discussion is not about the usual reflexive pronoun, but about a strange reflexive which must be explained from the standpoint of the "I". Grammarians call it the "indirect reflexive". (In Greek it is a separate form.) E.g. "When John Smith spoke of James Robinson, he spoke of his brother, but he did not know that."
So it is conceivable that someone does not know that the Object of which he speaks is himself.
Now, if "I" is compatible with ignorance, the reflexive pronoun cannot be used as usual.
Now one may ask: was the person of which Smith intended to speak not Smith? Was the person not himself?.
Answer: not in the relevant sense! Unless the reflexive pronoun is itself a sufficient proof of reference. And the usual reflexive pronoun cannot do that.
I 96
I/Self/Logic/Anscombe: here, the "manner of givenness" is unimportant.
Fra I 97
The logician understands that "I" in my mouth is just another name for "E.A.". His rule: if x makes assertions with "I" as the subject, then they are true iff the predicates of x are true.
AnscombeVsLogic/AnscombeVsKripke: for this reason he makes the transition from "I" to "Descartes".
But this is too superficial: If one is a speaker who says "I", then it is impossible to find out what it is that says "I". E.g. one does not look to see from which apparatus the noise comes.
Thus, we have to compel our logician to assume a "guaranteed" reference of "I".
Fra I 98
Problem: with a guaranteed reference there is no longer any difference between "I" and "A".

Anscombe I
G.E. M. Anscombe
"The First Person", in: G. E. M. Anscombe The Collected Philosophical Papers, Vol. II: "Metaphysics and the Philosophy of Mind", Oxford 1981, pp. 21-36
In
Analytische Theorien des Selbstbewusstseins, Manfred Frank Frankfurt/M. 1994

Fra I
M. Frank (Hrsg.)
Analytische Theorien des Selbstbewusstseins Frankfurt 1994
Lewis, D. Verschiedene Vs Lewis, D. Metz II 274
Nida-RümelinVsLewis: this objection is off the table here after we have shown that on the 1st level (Marianna finds a colorfully furnished room with partly wrongly colored prints) the alternatives come into view, which are then excluded on the 2nd level. Real phenomenal knowledge.
Lewis I 9
ShafferVsIdentity Theory: it cannot be true because experiences with analytical necessity are not spatial while neural events take place in the nervous system. LewisVsShaffer: this is not analytical or otherwise necessary. And neural events are also abstract. Whatever results from considerations about experiences as an argument for nonspatiality should also apply to neural events. - VsLewis: it is nonsense to consider a mere sound chain or character string as a possible carrier of a meaning or a truth value. Meaning/Carrier: Carriers of meaning are only single speech acts!
II 213
LewisVsVs: my assertion is not that sounds and characters are carriers of meaning, but that they carry meaning and truth relatively to a language or population. A single speech act can be the bearer of meaning because in most cases it unambiguously determines the language used in its particular enforcement situation. - VsLewis: A meaning theory recurred to a possible world is circular. - Def Possible World/VsLewis): The concept of a possible world can itself be explained by recourse to semantic terms. Possible worlds are models of the analytical propositions of a language or diagrams or theories of such models. -LewisVs: Possible world cannot be explained by recourse to semantic terms. Possible worlds exist and should not be replaced by their linguistic representations. 1. Such a substitution does not work properly: two worlds which are not different in the representing language get (wrongly) assigned to one and the same representation.
II 214 ++
2. Such a replacement would also be completely unnecessary: the concept of possible worlds is perfectly understandable in itself.
II 216
Hypostatization of meaning - VsLewis: not just words, things exist! - VsVs: we can form a grammar
II 221
VsLewis: maybe internal representation? VsVs: that does not help!
II 222
Convention is more than agreement: the others must believe in it!
II 223
VsLewis:Language conventions are no better than our infamous obscure old friends, the language rules. VsVs: A convention of truthfulness and trust could be called a rule.
II 224
VsLewis: Language is not conventional. LewisVs: There may be less conventionality than we originally thought. However, there are conventions of language.
II 225
VsLewis: Only those who are also set theorists can expect others to adhere to regularity. LewisVs: An ordinary person does not need to possess a concept of L in order to be able to expect that the others are truthful and trusting in L. He only needs to have expectations about action.
II 226
VsLewis: Using language is almost never a rational matter. LewisVs: An action can be rational and explainable even if it is done out of habit and without thought.
II 227
VsLewis: Language cannot possibly be traced back to conventions. It is impossible to agree on everything at any time. LewisVs: Admittedly, the first language cannot possibly go back to a convention.
II 227
VsLewis: E.g. Suppose a lifelong isolated person could one day spontaneously start using a language due to his ingenious talent. LewisVs: Even people living in isolation always adhere to a certain regularity.
II 228
VsLewis: It is circular to define the meaning in P of sentences using the assumptions made by the members of P. LewisVs: It may be so, but it does not follow that making an assumption should be analyzed as accepting sentences.
II 229
VsLewis: E.g. Suppose population of notorious liars. LewisVs: I deny that L is used in this population!
II 229
E.g. Ironist: these people are actually true in L! But they are not literally true in L! I.e. they are truly in another language, connected with L, which we can call "literal-L".
II 232
VsLewis: Truthfulness and trust (here not in L) cannot be a convention. LewisVs: The convention is not the regularity of truthfulness and trust par excellence. It is in a certain language! Its alternatives are regularities in other languages!
II 233 +
VsLewis: Even truthfulness and trust in L cannot be a convention. Moral obligation/Lewis: a convention continues to exist because everyone has reason to abide by it, if others do, that is the obligation. VsLewis: Why communication when people can draw completely different conclusions from a statement?
II 234
VsVs is quite compatible with my theory. But these are not independent conventions but by-products.
II 235
VsLewis: not only one language, but an infinite number of fragments (e.g. interest in communication etc.) VsVs: this is indeed the case, the language is inhomogeneous e.g. educated/uneducated.
II 237
VsLewis: silence is not untruthful. VsVs: Right expectation of truthfulness, but no trust!
II 238/239
VsLewis: either analytical or not, no smooth transition! VsVs: fuzzy analyticity with the help of gradual conventionality: regarding the strength of assumptions or the frequency of exceptions, or uncertainty as to whether certain worlds are actually possible.
II 240
VsLewis: thesis and anti-thesis refer to different objects: a) semantic (artificial) languages, b) language as part of natural history - VsVs: no, there is only one philosophy of language, language and languages are complementary!





Lewis I
David K. Lewis
Die Identität von Körper und Geist Frankfurt 1989

Lewis I (a)
David K. Lewis
An Argument for the Identity Theory, in: Journal of Philosophy 63 (1966)
In
Die Identität von Körper und Geist, Frankfurt/M. 1989

Lewis I (b)
David K. Lewis
Psychophysical and Theoretical Identifications, in: Australasian Journal of Philosophy 50 (1972)
In
Die Identität von Körper und Geist, Frankfurt/M. 1989

Lewis I (c)
David K. Lewis
Mad Pain and Martian Pain, Readings in Philosophy of Psychology, Vol. 1, Ned Block (ed.) Harvard University Press, 1980
In
Die Identität von Körper und Geist, Frankfurt/M. 1989

Lewis II
David K. Lewis
"Languages and Language", in: K. Gunderson (Ed.), Minnesota Studies in the Philosophy of Science, Vol. VII, Language, Mind, and Knowledge, Minneapolis 1975, pp. 3-35
In
Handlung, Kommunikation, Bedeutung, Georg Meggle Frankfurt/M. 1979

Lewis IV
David K. Lewis
Philosophical Papers Bd I New York Oxford 1983

Lewis V
David K. Lewis
Philosophical Papers Bd II New York Oxford 1986

Lewis VI
David K. Lewis
Convention. A Philosophical Study, Cambridge/MA 1969
German Edition:
Konventionen Berlin 1975

LewisCl
Clarence Irving Lewis
Collected Papers of Clarence Irving Lewis Stanford 1970

LewisCl I
Clarence Irving Lewis
Mind and the World Order: Outline of a Theory of Knowledge (Dover Books on Western Philosophy) 1991
Prosentential Theory Verschiedene Vs Prosentential Theory Horwich I 344
Quote/VsProsentential Theory/Camp, Grover, Belnap/VsCGB: one accuses the prosentential theory of ignoring cases where truth of quotes, i.e. names of sentences is stated. Example (27) "Snow is white" is true.
CGB: we could say here with Ramsey that (27) simply means that snow is white.
CGBVsRamsey: this obscures important pragmatic features of the example. They become clearer when we use a foreign-language translation. Example
(28) If „Schnee ist weiß“ is true, then…
Why (28) instead of
If it’s true that snow is white, then
Or
If snow is white, then…
CGB: there are several possible reasons here. We may want to make it clear that the original sentence was written in German. Or it could be that there is no elegant translation, or we do not know the Grammar of German well enough. Or example: "Snow is white" must be true because Fritz said it and everything Fritz says is true.
I 345
Suppose English* has a way of formally presenting a sentence: E.g. „Betrachte __“ („Consider____").
(29) Consider: Snow is white. This is true.
CGB: why should it not work the same as "snow is white is true" in normal English?
VsCGB: you could argue that it requires a reference to sentences or expressions because quotation marks are name-forming functors.
Quotation marks/CGB: we deviate from this representation! Quotation marks are not name-forming functors. ((s) not for CGB).
Quote/CGB: should not be considered as a reference to expressions in normal English. But we do not want to follow that up here.
I 346
VsCGB: one has accused the prosentential theory of tunnel vision: Maybe we overlooked certain grammatically similar constructions? Example (30) John: there are seven legged dogs
Mary: that's surprising, but true.
(31) John: the being of knowledge is the knowledge of being
Mary: that is profound and it is true.
Ad (30): of course the first half is "that is surprising" in no way prosentential. It is a characterization!
VsCGB: Ad (31) "is profound" expresses a quality that Mary attributes to the sentence. Why shouldn't "true" be understood in the same way?
CGB: it makes sense to take "this" here as referring to a sentence. But that would make things more complicated because then we would have to treat "that" and "it" differently in "that's true" and "it's true".
CGBVsVs: 1. it is just not true that the "that" in "that's surprising" refers to an utterance (in the sense of what was said, or a proposition).
What is surprising here? Facts, events or states of affairs.
Statement/Surprise/CGB: a statement can only be surprising as an act.
I 347
The surprising thing about the statement is the fact reported. ((s) But then the content rather than the act of testimony.)
CGBVs(s): it is not the fact that there are seven legged dogs claimed to be true in (30), because that fact cannot be true!
Proposition/CGB: (ad (31) Propositions are not profound. Acts can be profound. For example insights or thoughts.
Truth/Act/Action/Statement/CGB: but statements in the sense of action are not what is called true. ((s) see also StrawsonVsAustin, ditto).
Reference/Prosentential Theory/CGB: even if we consider "that's surprising, but it's true" as referring, the two parts don't refer to the same thing! And then the theory is no longer economic.
Reference/Prosentential Theory/CGB: are there perhaps other cases where it is plausible that a pronoun refers to a proposition? Example
(32) John: Some dogs eat grass.
Mary: You believe that, but it's not true.
Proposition: is often understood as a bearer of truth, and as an Object of belief. (CGBVs).
I 348
However, if "that" is understood here as a referencing pronoun, then the speaker must be a proposition. CGBVs: we can interpret "that you believe" also differently: as prosentential anaphora (as above in the example "that is wrong", with preceding negation prefix). Then we have no pronominal reference.
N.B.: the point is that no property is attributed. Truth is not a property.
VsCGB: another Objection: it is also a "tunnel vision" that we only have "that is true" but not "that is right" in view. Or the example "exaggerated" by Austin.
Example: a child says
I've got 15 logs
That is right.
I 349
Question: should this (and e.g. "This is an exaggeration!") be understood prosententially? CGBVsVs: "that is right" is here the statement that the child counted right, that it did something right. Sometimes this can overlap with the statement that a statement is true. The overlap must exist because there is no clear boundary between language learning and use.
I 349
Anaphora/Prosentential Theory/VsCGB: could not one split the prosody and take the individual "that" as an anaphora? CGBVsVs: then one would also have to split off "is true" and no longer perceive it as referencing, but as characterizing ((s) And thus attributing it as property).
CGBVs: then we would have to give up our thesis that speech about truth is completely understandable without "carrier of truth" or "truth characteristic".
Moreover:
Reference/CGB: it is known that not every nominalization has to be referencing ((s) E.g. Unicorn).
Predication/CGB: also not every predication has to be characterizing.
Divine Perspective/outside/PutnamVsGod's point of view/Rorty: Putnam amuses himself like James and Dewey, about such attempts.
Rorty: But he has a problem when it comes to PutnamVsDisquotationalism: it smells too reductionist, too positivist, too "behaviorist" ("transcendental skinnerism").
Truth/Putnam: when a philosopher says truth is something other than electricity because there is room for a theory of electricity but not for a truth theory,
I 456
and that knowledge of the truth condition is all that could be known about truth, then he denies that truth is a property. So there is also no property of correctness or accuracy ((s) >Deflationism, PutnamVsDeflationism, PutnamVsGrover.) PutnamVs: that is, to deny that our thoughts are thoughts and our assertions are assertions.
Theory/Existence/Reduction/Putnam/Rorty: Putnam here assumes that the only reason to deny is that you need a theory for an X is to say that the X is "nothing but Y" ((s) eliminative reductionism).
PutnamVsDavidson: Davidson must show that claims can be reduced to sounds. Then the field linguist would have to reduce actions to movements.
Davidson/Rorty: but this one does not say that claims are nothing but sounds.
Instead:
Truth/Explanation/Davidson: other than electricity, truth is no explanation for something. ((s) A phenomenon is not explained by the fact that a sentence that claims it is true).





Horwich I
P. Horwich (Ed.)
Theories of Truth Aldershot 1994
Ramsey, F. P. Grover Vs Ramsey, F. P. Horwich I 319
VsRedundancy Theory/VsRamsey/Camp, Grover, Belnap/CGB/Grover: the first two objections assume that the data base is too narrow, i.e. that there are cases that are not covered by the theory. (See Redundancy Theory).
I 320
1)
Index words: (Here: repetition of indices): (14) John: I’m greedy - Mary: That is true Problem: here no mere repetition, or else she would say "I am,..." Problem: there is no general scheme for such cases. 2)
Modification: Here, a translation is absolutely impossible: (here with indirect reference and quantification):
(15) Every thing that Mark said could be true Problem: there is no verb for "could". Similar:
(16) Something that Charlie said is either true or not true.
(17) Everything that Judith said was true then, but none of it is still true today. Of course you can try:
(15’)(p) Mark said that p > It could be the case that p) or
(15’)(p) (Mark said that p > that might p exist) Vs: "being the case" and "existing" are variations of "being true". This would make the redundancy theory a triviality. In this case, Ramsey’s "direct" theory would be wrong. CGBVsRamsey: we improve the redundancy theory by we let by not only allowing propositional quantification for the target language, but also an indeterminate field of links, such as M (for "might"), "P" (for past tense), "~" for negation, etc.
I 321
The reader has likely already assumed that we have introduced the negation long ago. But that’s not true. Then: (16’)(p) (Mark said that p > Mp)
(17’)(Ep) (Charlie said that p & (p v ~p))
(17’)(p) (Judith said that p > (Pp & ~p))
Redundancy Theory/Ramsey/CGB: it is this variant of the theory of Ramsey, enriched by the above links and propositional quantification, which we call redundancy theory (terminology) from now on. The thesis is that "true" thus becomes superfluous. Thesis this allows translations in Ramseyan sense to be found always.
VsRedundancy Theory/VsRamsey: 3) "About"/Aboutness/Accuracy of the Translation/CGB: some authors: argue that "snow is white" is about snow, and "That snow is white, is true" is about the proposition. And that therefore the translation must fail.
CGB: this involves the paradox of analysis. We do not directly touch upon it. ((s) Paradox of analysis, here: you’d have to act more stupid than you are in order not to realize that both sentences are about snow; to be able to name the problem at all (as the opponents do) you need to have it solved already.)
4)
PragmatismVsRedundancy Theory: even if the translation preserves the alleged content, it neglects other features which should be preserved. Case of recurrence: E.g.
(3) Mary: Snow is white. John: That is true.
(3’) Mary: Snow is white. John: Snow is white. Is that supposed to be a good translation?.
I 322
Strawson: "true" and "not true" have their own jobs to do!. Pro-Sentence/Pronoun/Anaphora/"True"/CGB: "that is true" presupposes that there is an antecedent. But that is not yet taken into account in Ramsey’s translation (3’). So Ramsey’s translation fails in pragmatic terms.
VsPropositional Quantification/PQ/VsRedundancy Theory/VsRamsey/CGB: 4) redundancy: at what price? Propositional quantification is mysterious: it is not consistent with everyday language. It is not shown that "is true" is superfluous in German, but only in a curious ad hoc extension. 5) Grammar: (already anticipated by Ramsey): variables need predicates that are connected with them, even if these variables take sentence position. CGBVsRamsey: unfortunately, Ramsey’s response is not convincing. Ramsey: (see above) "p" already contains a (variable) verb. We can assume the general sentence form as aRb here, then.
I 232
(a)(R)(b): If he says aRb, then aRb). Here,"is true" would be a superfluous addition. CGBVsRamsey: We must assume an infinite number of different sentence forms ((s)> language infinite). Redundancy Theory/CGB: But that does not need to worry us. 1) Propositional quantification can be set up formally and informally proper. 2) Variables which take sentences as substituents do not need a verb that is connected to them. That this was the case, is a natural mistake which goes something like this:
E.g.(4’) (p)(John says p > p).
If we use pronouns that simplify the connected variable:
For each sentence, if John said it, it then it.
Heidelberger: (1968): such sentences have no essential predicate!.
Solution/Ramsey:
(4’) For each sentence, if John said that it is true, then it is true. T-Predicate/CGB: "T": reads "is true".
(4’) (p) (John said that Tp > Tp) Problem: because "T" is a predicate, and "Tp" is a sentence, "p" must be a term of the language, i.e. it must take a nominal position. I.e. the quantifiers bind individual variables (of a certain type), and not variables about sentences.
I 335
Disappearance Cases/Pro-Sentence: some of them can be regarded as a translation in Ramsey language. Def Ramsey Language/CGB/(s): Language in which "true" is entirely superfluous. English*/CGBVsRamsey: for the purpose of better explanation. E.g. (26) It is true that snow is white, but in Pittsburgh it rarely looks white.
(27) It is true that there was unwarranted violence by the IRA, but it is not true that none of their campaigns was justified. T-Predicate/CGB: used in (25) and (26) to concede a point in order to determine afterwards by "but" that not too much emphasis should be placed on it. English*.
I 336: E.g.
(26’) There was unwarranted violence by the IRA, that’s true, but it is not true that none of their campaigns was justified. These are all disappearance cases.
I 342
VsProsentential Theory/Spurious Objections/CGB:
I 343
Index Words: Laziness pro-sentences refer to their antecedent. Therefore, the theory must be refined further when it comes to indexical expressions. Otherwise E.g. John: "I’m lazy." Mary: "That’s true." Is not to say that Mary means "I (Mary) am lazy". CGB: but that’s a common problem which occurs not only when speaking about truth: E.g. John: My son has a wart on his nose. Bill: He is the spitting image of his father. E.g. Lucille: You dance well. Fred: That’s new to me. Pragmatics/CGBVsRamsey: our approach represents it correctly, in particular, because we exclude "plagiarism". Ramsey’s theory does not.
I 344
Quote/VsPro-Sentence Theory/VsCGB: The pro-sentence theory is blamed to ignore cases where truth of quotes, i.e. names of sentences, is expressed. E.g. (27) "Snow is white" is true. CGB: We could say with Ramsey, that (27) simply means that snow is white. CGBVsRamsey: that obscures important pragmatic features of the example. They become more apparent when we use a foreign language translation. E.g.
(28) If "snow is white" is true, then... Why (28) instead of If it’s true that snow is white, then or If snow is white, then... CGB: There are several possible reasons for this. It may be that we want to make clear that the original sentence was said in German. Or it is possible that there is no elegant translation, or we are not sufficiently familiar with German Grammar. Or E.g. "snow is white" must be true, because Fritz said it, and everything Fritz says is true.
I 345
Suppose, English* had a possibility to present a sentence formally: E.g. "consider __".
(29) Consider: Snow is white. This is true. CGB: why should it not work just like "Snow is white is true" in normal English? VsCGB: it could be argued that this requires a reference on sentences or expressions, because quotation marks are name-forming functors. Quotation Marks/CGB: we depart from this representation! Quotation marks are not name-forming functors.
I 353
Propositional Variable/Ramsey: Occupies sentence position. (Quantification over propositions). CGBVsRamsey: Such variables are of pro-sentential nature. Therefore, they should not be connected to a T-predicate. ((s) otherwise, "true" appears twice). T-Predicate/Ramsey/Redundancy Theory/CGB: this answers the old question of whether a Ramsey language has to contain a T-predicate: see below. Our strategy is to show how formulas can be read in English*, where there is no separable T-predicate. E.g. (4’) For each proposition, if John says it is true, then it is true. CGB: in this case,propositional variables and quantificational pro-sentences do the same job. Both take sentence position and have the cross-reference that is required of them. Important argument: (4’) is just the candidate for a normal English translation of (4’). Problem: this could lead to believing that a Ramsey language needs a T-predicate, as in
(4’) (p)(John said that Tp > Tp). ((s) then, "true" implicitly appears twice).
I 354
But since (4’) is perfect English, there is no reason to assume that the T-predicate is re-introduced by that. Or that it contains a separately bound "it" (them).

Grover I
D. L. Grover
Joseph L. Camp
Nuel D. Belnap,
"A Prosentential Theory of Truth", Philosophical Studies, 27 (1975) pp. 73-125
In
Theories of Truth, Paul Horwich Aldershot 1994

Horwich I
P. Horwich (Ed.)
Theories of Truth Aldershot 1994
Sense Data Sellars Vs Sense Data I 9
SellarsVsSense Data Theory: mistake: as with the naturalistic fallacy: to consider the reality as a fact that requires no learning! It looks indeed strange that one would have to learn a sensation of pain or sensation of color. But if the sensation is not learned, then the theorists cannot perform any analysis that assumes the acquired skills. But a classificatory distinction does not work without learning and without conceptualization, or even without the use of symbols.
I 10
SellarsVsSense Data Theory: three assumptions are contrary to each other: A. The proposition X perceives a red sense content s stating that X in a non-inferential way knows that s is red.
B. The ability to feel the meaning content is not learned.
C. The ability to know facts of the form x is φ is learned.
A and B together contain non-C; B and C contain non-A; A and C contain non-B. ((s) ratio of three propositions that in pairs respectively exclude the third.).
Three possibilities:
(1) you can drop A. Then the sensation becomes a noncognitive fact. This can, of course, build a necessary condition, even a logically necessary condition for a non-inferential knowledge.
I 11
(2) One can drop B. Thus the concept of the sense data is detached from our everyday speech on sensations, feelings, itching. (3) to drop C would, however, be contrary to the nominalistic trends that were prevalent within the empiricist tradition.
Sense data/Sellars: the concept of the sense data seems to be a hybrid of two ideas:
1. the idea that there are certain inner episodes as red sensations, without a process of learning or conceptualization would have preceded. Without these inner episodes one could in some way not see!
2. the idea that there are certain inner episodes that are non-inferential content of knowledge. These episodes are necessary conditions of empirical knowledge as evidence base ("evidence") for all other empirical claims.
I 12
Right now, it does not follow that the sensation of a red triangle is a cognitive or epistemic fact. You are of course tempted to equate "To have the sensation of a red triangle" with the "Thinking of a heavenly city", then the former is epistemic and intentional, the latter only intentional. But this temptation can be resisted. Because you can claim that the sensation is a fact sui generis that is neither epistemic nor physically and that has its own logical Grammar! Unfortunately, that was often associated with a false reasoning:
False: we might describe "seeing that a facing side of a physical Object is red and is triangular," as "apparent act of seeing" of which some are not reliable. From a subjective perspective there is no indicator which ensures that any such information is reliable! By more precise information on the circumstances a class some more reliable observations can be created. But no complete reliability.
I 13
Sellars: that confuses a lot: we remember that the sensations of red triangles have exactly those advantages that they are missing the apparent acts of seeing physical surfaces. From the analogy of sensations with the "thinking of a heavenly city", one might think that sensations were in the same category as thoughts. So that both are cognitive facts. Then you will find that sensations are much closer to mental processes than external physical Objects.
Mistake: to overlook the fact that one can only describe an experience as reliable when
it is also useful to refer to it as unreliable.
I 24
To appear/to seem/theory of the appearance/Sellars: VsSense data theory: assume that the facts of the form "x seems to be φ for S"
are atomically and irreducible and that you need sense data neither for its analysis nor for an explanation of these facts. (Sellars pro). The proposition that something seems to be red for someone who has the idea that he is in any relationship with something that is red, not as part of its meaning!
Sellars: to seem prima facie = to be.

Sellars I
Wilfrid Sellars
The Myth of the Given: Three Lectures on the Philosophy of Mind, University of London 1956 in: H. Feigl/M. Scriven (eds.) Minnesota Studies in the Philosophy of Science 1956
German Edition:
Der Empirismus und die Philosophie des Geistes Paderborn 1999

Sellars II
Wilfred Sellars
Science, Perception, and Reality, London 1963
In
Wahrheitstheorien, Gunnar Skirbekk Frankfurt/M. 1977
Stalnaker, R. Lewis Vs Stalnaker, R. Read III 101/102
Stalnaker equates the probability of the conditional clauses with the conditional probability. LewisVsStalnaker: there is no statement whose probability is measured by the conditional probability! (+ III 102)
According to Lewis, based on Stalnaker's assumption, the odds of drawing cards are independent. But this is obviously wrong (as opposed to throwing dice). Thus, the probability of the conditional clause cannot be measured by the conditional probability.
III 108
Example from Lewis If Bizet and Verdi were compatriots, Bizet would be Italian.
and
If Bizet and Verdi were compatriots, Bizet wouldn't be Italian.
Stalnaker: one or the other must be true.
Lewis: both are wrong. (Because only subjunctive conditional sentences are not truth functional). The indicative pieces would be entirely acceptable to those who do not know their nationality.

Lewis IV 149
Action/Rationality/Stalnaker: Propositions are the suitable objects of settings here. LewisVsStalnaker: it turns out that he actually needs a theory of attitudes de se.
Stalnaker: the rationally acting is someone who accepts various possible rational futures. The function of the wish is simple to subdivide these different event progressions into the desired and the rejected ones.
Or to provide an order or measure of alternative possibilities in terms of desirability.
Belief/Stalnaker: its function is simple to determine which the relevant alternative situations may be, or to arrange them in terms of their probability under different conditions.
Objects of attitude/Objects of belief/Stalnaker: are identical if and only if they are functionally equivalent, and they are only if they do not differ in any alternative possible situation.
Lewis: if these alternative situations are always alternative possible worlds, as Stalnaker assumes, then this is indeed an argument for propositions. ((s) Differentiation Situation/Possible world).
Situation/Possible world/Possibility/LewisVsStalnaker: I think there can also be alternatives within a single possible world!
For example, Lingens now knows almost enough to identify himself. He's reduced his options to two: a) he's on the 6th floor of the Stanford Library, then he'll have to go downstairs, or
b) he is in the basement of the Widener College library and must go upstairs.
The books tell him that there is exactly one person with memory loss in each of these places. And he found out that he must be one of them. His consideration provides 8 possibilities:
The eight cases are spread over only four types of worlds! For example, 1 and 3 do not belong to different worlds but are 3000 miles away in the same world.
In order to distinguish these you need qualities again, ((s) the propositions apply equally to both memory artists.)
V 145
Conditionals/Probability/Stalnaker: (1968)(1) Notation: ">" (pointed, not horseshoe!) Def Stalnaker Conditional: a conditional A > C is true if and only if the least possible change that makes A true, also makes C true. (Revision).
Stalnaker: assumes that P(A > C) and P(C I A) are adjusted if A is positive.
The sentences, which are true however under Stalnaker's conditions, are then exactly those that have positive probabilities under his hypothesis about probabilities of conditionals.
LewisVsStalnaker: this is probably true mostly, but not in certain modal contexts, where different interpretations of a language evaluate the same sentences differently.
V 148
Conditional/Stalnaker: to decide whether to believe a conditional: 1. add the antecedent to your set of beliefs,
2. make the necessary corrections for the consistency
3. decide if the consequence is true.
Lewis: that's right for a Stalnaker conditional if the fake revision is done by mapping.
V 148/149
LewisVsStalnaker: the passage suggests that one should pretend the kind of revision that would take place if the antecedens were actually added to the belief attitudes. But that is wrong: then conditionalisation was needed.
Schwarz I 60
Counterpart/c.p./counterpart theory/c.p.th./counterpart relation/c.p.r./StalnakerVsLewis: if you allow almost arbitrary relations as counterpart relations anyway, you could not use qualitative relations. (Stalnaker 1987a)(2): then you can reconcile counterpart with Haecceitism: if you come across the fact that Lewis (x)(y)(x = y > N(x = y) is wrong, (Lewis pro contingent identity, see above) you can also determine that a thing always has only one counter part per world. Stalnaker/Schwarz: this is not possible with qualitative counterpart relations, since it is always conceivable that several things - for example in a completely symmetrical world - are exactly the same as a third thing in another possible world.
LewisVsStalnaker: VsNon qualitative counter part relation: all truths including modal truths should be based on what things exist (in the real world and possible worlds) and what (qualitative) properties they have (>"mosaic": >Humean World).
Schwarz I 62
Mathematics/Truthmaking/Fact/Lewis/Schwarz: as with possible worlds, there is no real information: for example, that 34 is the root of 1156, tells us nothing about the world. ((s) That it applies in every possible world. Rules are not truthmakers). Schwarz: For example, that there is no one who shaves those who do not shave themselves is analogously no information about the world. ((s) So not that the world is qualitatively structured).
Schwarz: maybe we'll learn more about sentences here. But it is a contingent truth (!) that sentences like "there is someone who shaves those who do not shave themselves" are inconsistent.
Solution/Schwarz: the sentence could have meant something else and thus be consistent.
Schwarz I 63
Seemingly analytical truth/Lewis/Schwarz: e.g. what do we learn when we learn that ophthalmologists are eye specialists? We already knew that ophthalmologists are ophthalmologists. We have experienced a contingent semantic fact. Modal logic/Modality/Modal knowledge/Stalnaker/Schwarz: Thesis: Modal knowledge could always be understood as semantic knowledge. For example, when we ask if cats are necessary animals, we ask how the terms "cat" and "animal" are to be used. (Stalnaker 1991(3),1996(4), Lewis 1986e(5):36).
Knowledge/SchwarzVsStalnaker: that's not enough: to acquire contingent information, you always have to examine the world. (Contingent/Schwarz: empirical, non-semantic knowledge).
Modal Truth/Schwarz: the joke about logical, mathematical and modal truths is that they can be known without contact with the world. Here we do not acquire any information. ((s) >making true: no empirical fact "in the world" makes that 2+2 = 4; Cf. >Nonfactualism; >Truthmakers).
Schwarz I 207
"Secondary truth conditions"/truth conditions/tr.cond./semantic value/Lewis/Schwarz: contributing to the confusion is that the simple (see above, context-dependent, ((s) "indexical") and variable functions of worlds on truth values are often not only called "semantic values" but also as truth conditions. Important: these truth conditions (tr.cond.) must be distinguished from the normal truth conditions.
Lewis: use truth conditions like this. 1986e(5),42 48: for primary, 1969(6), Chapter V: for secondary).
Def Primary truth conditions/Schwarz: the conditions under which the sentence should be pronounced according to the conventions of the respective language community.
Truth Conditions/Lewis/Schwarz: are the link between language use and formal semantics, their purpose is the purpose of Grammar.
Note:
Def Diagonalization/Stalnaker/Lewis/Schwarz: the primary truth conditions are obtained by diagonalization, i.e. by using world parameters for the world of the respective situation (correspondingly as time parameter the point of time of the situation etc.).
Def "diagonal proposition"/Terminology/Lewis: (according to Stalnaker, 1978(7)): primary truth conditions
Def horizontal proposition/Lewis: secondary truth condition (1980a(8),38, 1994b(9),296f).
Newer terminology:
Def A-Intension/Primary Intension/1-Intension/Terminology/Schwarz: for primary truth conditions
Def C-Intension/Secondary Intension/2-Intension/Terminology/Schwarz: for secondary truth conditions
Def A-Proposition/1-Proposition/C-Proposition/2-Propsition/Terminology/Schwarz: correspondingly. (Jackson 1998a(10),2004(11), Lewis 2002b(12),Chalmers 1996b(13), 56,65)
Def meaning1/Terminology/Lewis/Schwarz: (1975(14),173): secondary truth conditions.
Def meaning2/Lewis/Schwarz: complex function of situations and worlds on truth values, "two-dimensional intention".
Schwarz: Problem: this means very different things:
Primary truth conditions/LewisVsStalnaker: in Lewis not determined by meta-linguistic diagonalization like Stalnaker's diagonal proposition. Not even about a priori implication as with Chalmer's primary propositions.
Schwarz I 227
A posteriori necessity/Metaphysics/Lewis/Schwarz: normal cases are not cases of strong necessity. One can find out for example that Blair is premier or e.g. evening star = morning star. LewisVsInwagen/LewisVsStalnaker: there are no other cases (which cannot be empirically determined).
LewisVs Strong Need: has no place in its modal logic. LewisVs telescope theory: possible worlds are not like distant planets where you can find out which ones exist.


1. Robert C. Stalnaker [1968]: “A Theory of Conditionals”. In Nicholas Rescher (ed.), Studies
in Logical Theory, Oxford: Blackwell, 98–112
2.Robert C. Stalnaker [1987a]: “Counterparts and Identity”. Midwest Studies in Philosophy, 11: 121–140. In [Stalnaker 2003]
3. Robert C. Stalnaker [1991]: “The Problem of Logical Omniscience I”. Synthese, 89. In [Stalnaker 1999a]
4. Robert C. Stalnaker — [1996]: “On What Possible Worlds Could Not Be”. In Adam Morton und Stephen P.
Stich (Hg.) Benacerraf and his Critics, Cambridge (Mass.): Blackwell. In [Stalnaker 2003]
5. David Lewis [1986e]: On the Plurality of Worlds. Malden (Mass.): Blackwell
6. David Lewis[1969a]: Convention: A Philosophical Study. Cambridge (Mass.): Harvard University
Press
7. Robert C. Stalnaker [1978]: “Assertion”. In P. Cole (ed.), Syntax and Semantics, Vol. 9, New York: Academic Press, 315–332, und in [Stalnaker 1999a]
8. David Lewis [1980a]: “Index, Context, and Content”. In S. Kanger und S. ¨Ohmann (ed.), Philosophy
and Grammar, Dordrecht: Reidel, und in [Lewis 1998a]
9. David Lewis [1994b]: “Reduction of Mind”. In Samuel Guttenplan (ed.), A Companion to the Philosophy
of Mind, Oxford: Blackwell, 412–431, und in [Lewis 1999a]
10. Frank Jackson [1998a]: From Metaphysics to Ethics: A Defence of Conceptual Analysis. Oxford: Clarendon Press
11. Frank Jackson [2004]: “Why We Need A-Intensions”. Philosophical Studies, 118: 257–277
12. David Lewis [2002b]: “Tharp’s Third Theorem”. Analysis, 62: 95–97
13. David Chalmers [1996b]: The Conscious Mind. New York: Oxford University Press
14. David Lewis [1975]: “Languages and Language”. In [Gunderson 1975], 3–35. And in [Lewis 1983d]

Lewis I
David K. Lewis
Die Identität von Körper und Geist Frankfurt 1989

LewisCl I
Clarence Irving Lewis
Mind and the World Order: Outline of a Theory of Knowledge (Dover Books on Western Philosophy) 1991

Re III
St. Read
Thinking About Logic: An Introduction to the Philosophy of Logic. 1995 Oxford University Press
German Edition:
Philosophie der Logik Hamburg 1997

Schw I
W. Schwarz
David Lewis Bielefeld 2005
Tractatus Wittgenstein Vs Tractatus Tugendhat I 163
Tractatus/Tugendhat: naive object-theoretical position. Wittgenstein: "what the case, the fact is, is the existence of atomic facts", "the fact is a combination of objects". "In the facts objects hang one in another, like the links of a chain". (2.03). (Later discarded by Wittgenstein). Wittgenstein/late/self-criticism/VsTractatus: Philosophical remarks: "complex is not the same as fact I say of a complex, it is moving from one place to another, but not from a fact." "To say that a red circle consists of redness and circularity, or a complex of these constituents, is an abuse of such words and misleading."
---
I 235 ff
WittgensteinVsWittgenstein/WittgensteinVsTractatus/Hintikka. WWK, 209 f. "unclear to me in the Tractatus was the logical analysis and ostensive definition" ... "thought at this time that it is a connection between language and reality"... ---
I 236
Sign/Meaning/Definition/showing/Waismann ("theses"): "We can give meaning to characters in two ways:. 1. by designation 2. by definition". ---
I 237
Hintikka: deeper reasons: in the Tractatus the thesis of inexpressibility of semantics does not stop Wittgenstein from highlighting the role of the ostensive definition under the guise of showing. Through his move from phenomenology to the physical language it is impossible for him to indicatively define all his not further-back-tracable objects. One and the same gesture may be in the game when one indicatively defines a person's name, a color word, a substance name (mass terminus) a numeral, the name of a compass direction.
The differences apparantly do not seem to belong to the area of the phenomenological, but to the ontology of everyday Objects. Philosophical Investigations, PI § 28
For these reasons, Wittgenstein rejects for some time the idea that the ostensive explaining could establish a connection between language and reality.
---
I 297 ff
Image/agreement/reality/Wittgenstein/Hintikka: is the vividness an agreement? ---
I 298
Image/sentence/WittgensteinVsTractatus/WittgensteinVsWittgenstein/self-criticism: in the Tractatus I said something like: it was an agreement of the form, however, this is a mistake. Hintikka: this could give the wrong impression, that Wittgenstein abandoned the image thoughts. But that is a mistake.
Image/Wittgenstein: the image can represent a possible state of affairs. It does not need to be an image of a de facto state in the world. A command is usually an image of the action that should be performed, but not necessarily an image of the actual completed act. (Also work drawing).
What is the method of projection?
---
I 299
"So I imagine the difference between sentence and reality is offset by the projection beams belonging to the image, the idea and which leave no more room for a method of application. There is only agreement and disagreement." "Like everything metaphysical the harmony between thought and reality in Grammar can be found in the language."
---
II 138
Atomism/VsAtomism/self-criticism/WittgensteinVsTractatus: it was a mistake, that there were elementary propositions, into which all sentences can be dismantled. This error has two roots: 1. that one conceives of infinity as a number, and assumes that there is an infinite number of sentences.
2. statements that express degrees of qualities. ((s) they must not exclude any other sentence. Therefore, they cannot be independent).
---
III 151
Tractatus/later self-criticism/WittgensteinVsTractatus/WittgensteinVsWittgenstein: he was dealing with two weak points: 1. that the descriptive language is so openly regarded as a model of the actual language. There are many unrecognized forms of speech.
It may be questioned whether the meaning of an utterance can be understood regardless of the context. In addition, doubt, as to whether any meaningful sentence has one and only one logical form.
2. Problem of intersubjectivity disregarded.
---
III 214
WittgensteinVsTractatus (self-criticism): discussions with Ramsey and the Italian economy scientist Piero Sraffa. SraffaVsTractatus: VsImage theory: Vs, that a meaningful sentence must be a projection of a state of affairs. Also denied that any meaningful sentence could be resolved into elementary propositions.
From this critique emerged in 1929 30 Philosophical remarks (PB)
1932 34 Philosophical Grammar (PG)
1933 34 The Blue Book + The Brown Book
Main work of the "Second Period": Philosophical Investigations (Philosophical Investigations).
---
III 217
WittgensteinVsTractatus/Wittgenstein/late/Flor: that can be useful and clear in a specific situation, to give a vague question or a vague description or a vague instruction. ---
VI 95/96
Logical constants/elementary proposition/WittgensteinVsTractatus/WittgensteinVsWittgenstein/Schulte: self-criticism: does now no longer assume that one would be able later to specify elementary propositions. In truth, we already have everything, namely at present.
      New: Priority of sentence system over the individual sentence.
      Previously: I believed that we have to do without the logical constants, because "and", "or", "not" do not connect the Objects. (I abide by this).
      But I falsely believed that the elementary propositions would be independent from each other because I falsely believed the linking rules of logical constants could have something to do with the internal structure of sentences.
In reality, the logical constants form rather just a part of a comprehensive syntax of which I did not know anything then."
---
VII 148
Language/Tractatus/Tetens: language only serves one purpose here: to map facts. WittgensteinVsWittgenstein/VsTractatus/later Wittgenstein/Tetens: instead there is a variety of language games. To speak sensibly, we must take part in a complicated social life form with its diverse language games.
---
VII 149
The philosopher must describe how we use the expressions in everyday language. ---
VII 150
"... a picture holds us captive. And we could not get out because it was in our language, and it seemed to repeat it to us inexorably." (Philosophical Investigations, PI 82) Descriptive/normative/Tractatus/Tetens: Wittgenstein's ignores in the Tractatus the distinction between descriptive and normative sentences. He later calls this the "one-sided diet" ((s) only descriptive sentences). (Philosophical Investigations, PI p. 251, § 593)
---
VII 152
Skepticism/philosophy/Wittgenstein/late: also the philosophers learned the words "error", "doubt", etc., from the everyday language, they have not been invented for the purpose of philosophizing. ---
VII 153
Deception/Wittgenstein/late: when the philosopher asks if one could not be mistaken about everything, then he uses the words in a way that he would never use them in everyday life. ---
VII 154
Wittgenstein: E.g. one cannot say that one his mistaken about something in his joy.

W II
L. Wittgenstein
Wittgenstein’s Lectures 1930-32, from the notes of John King and Desmond Lee, Oxford 1980
German Edition:
Vorlesungen 1930-35 Frankfurt 1989

W III
L. Wittgenstein
The Blue and Brown Books (BB), Oxford 1958
German Edition:
Das Blaue Buch - Eine Philosophische Betrachtung Frankfurt 1984

W IV
L. Wittgenstein
Tractatus Logico-Philosophicus (TLP), 1922, C.K. Ogden (trans.), London: Routledge & Kegan Paul. Originally published as “Logisch-Philosophische Abhandlung”, in Annalen der Naturphilosophische, XIV (3/4), 1921.
German Edition:
Tractatus logico-philosophicus Frankfurt/M 1960

Tu I
E. Tugendhat
Vorlesungen zur Einführung in die Sprachanalytische Philosophie Frankfurt 1976

Tu II
E. Tugendhat
Philosophische Aufsätze Frankfurt 1992
Type Theory Wittgenstein Vs Type Theory II 439
Type Theory/Theory of types/WittgensteinVsRussell: f(a) = U's coat is red
F(a) = U's coat has one of the colors of the rainbow
φ(f) = Red is a color of the rainbow
Question: Now, φ (F) has a meaning? ((s) This is not mentioned in this combination above).
Russell: would say that "a color of the rainbow has the property to be a rainbow color" has no meaning, so that "f(f)" generally has no meaning.
But if we now create a rule of Grammar in order to exclude a replacement option (and exactly this does the theory of types, in order to avoid contradictions), then we must make the replacement rule dependendet exclusively on the characteristics of symbols.
Replacement rule: if we introduce "f(x)" we must not give "f (f)" a meaning.
E.g. Consider ~ f(f) = F(f) and the expression that is obtained by replacing "f" through "F": the property to not have oneself as a property that has itself in turn as a property. The root of the contradiction is that one considers a function to function of itself. ((s)> heterology).
From ~ f(f) = F(f) results the contradiction F(F) = ~ F(F).
Problem: arises when one declares a function to its function of itself.
II 440
"f" in "f(x)" cannot be used as an argument itself. But why should this not occur as that which one presupposes, is not a sentence? It is not true to say that here the principle of contradiction has been violated, because that could only be the case if one was talking about sentences.
Hardy said it would be unbearable to have real numbers of different orders.
See his discussion, after which a sequence of real numbers belongs to another order, because it is defined by reference to a entirety whose barrier it is itself.
An analog example is the maximum of a curve, which is defined as the highest points of all on this curve.

IV 68
Operation/Form Series/Type theory/TT/Tractatus: 5.252 only like this the progression from member to member in a form series (from type to type in Russell) is possible. WittgensteinVsRussell: in Principia Mathematica(1) (PM), they have not given the possibility of this progression, but have made use of it repeatedly.
5.2521 The repeated application of an operation to its own result ((s)> recursion) I call its successive application ("O'O '=' a" is the result of a triple application of "O'ζ" to "a").
5.2522 the general term of a form series a, O 'a, O'O'a ... I write:
IV 69
"[a,x,O'x]". This expression in brackets is a variable.
1. member: beginning of the form series
2. member: The form of any member x of the series
3. member: Form of the immediate successor this x. (Successor: O').
IV 70
WittgensteinVsRussell/Tractatus: 5.4 "Logical objects" or "logical constants" in the sense of Russell do not exist. Primitive signs/WittgensteinVsFrege/WittgensteinVsRussell/Tractatus: 5.42 The possibility of crosswise definition of the logical "primitive signs" of Frege and Russell (e.g. >, v) already shows that these are not primitive signs, let alone that they do not signify any relations.


1. Whitehead, A.N. and Russel, B. (1910). Principia Mathematica. Cambridge: Cambridge University Press.

W II
L. Wittgenstein
Wittgenstein’s Lectures 1930-32, from the notes of John King and Desmond Lee, Oxford 1980
German Edition:
Vorlesungen 1930-35 Frankfurt 1989

W IV
L. Wittgenstein
Tractatus Logico-Philosophicus (TLP), 1922, C.K. Ogden (trans.), London: Routledge & Kegan Paul. Originally published as “Logisch-Philosophische Abhandlung”, in Annalen der Naturphilosophische, XIV (3/4), 1921.
German Edition:
Tractatus logico-philosophicus Frankfurt/M 1960
Various Authors Mackie Vs Various Authors Stegmüller IV 399
"Kalam" argument: (common among Islamic scholars): operates with paradoxes of infinity to show that there can be no actual infinity. (> Al Ghassali). Infinity/MackieVsKalam argument: the possibility of an unlimited past cannot be ruled out on purely logical grounds!
MackieVsKant: this prejudice can also be found in the thesis about the first antinomy.
IV 400
Kalam argument/Al Ghassali: nothing that comes into existence in time, arises out of itself. ("Rational necessity"). Therefore, a creator is required. MackieVsAl Ghassali: 1. do we really know that from necessity of reason?
2. There is no reason why on one hand an uncaused thing should be impossible, but on the other hand the existence of a God with the power to create out of nothing, should be acceptable!
God/Mackie/Islam: this concept of God raises difficult problems:
1. Has God simply emerged with the time?
2. Has he always existed in infinite time? Then the formerly rejected actual infinity would be reintroduced!
3. Does God have a non-temporal existence: that would be an incomprehensible mystery again.
Mackie: additionally, one also has to assume:
a) that God's existence and creative power explain themselves and
b) that the unexplained existence of a material world would be incomprehensible and therefore unacceptable.

IV 401
Existence/MackieVsLeibniz: there is no reason a priori to indicate that things do not just occur without causation! Cosmology/proof of the existence of God/existence/Mackie: problem: either the notion of "causa sui" makes sense or not.
a) it does not make sense: then the cosmological assumption that a divine cause must be assumed for the beginning of material existence collapses.
b) it makes sense: then it can even be awarded as a property to matter itself!

Stegmüller
IV 447
Def. God/Feuerbach: "God is the sense of self of human kind freed from all loathsomeness." Religion/Feuerbach: utopia of a better religion: God's freedom from all limitations of individuals that was imputed by traditional religions now recovered in humanity as a whole.
MackieVsFeuerbach: humanity as a whole is undoubtedly not free from all limitations of individuals, it is not omnipotent, not omniscient, not all good. (vide supra: entirety as a wrong subject, cannot even act.

IV 472
Theodicy/faith/Stegmüller: Argument: God has made the earth a vale of tears, so that people would develop a religious need. MackieVs: only a very human deity could want people so submissive.
Theodicy/Gruner: insinuates to skeptics the demand for a world that is liberated from all evils. He rejects this demand as inconsistent.
MackieVsGruner: shifts the burden of proof. The skeptic demands nothing at all.

IV 271
Ethics/Education/Rousseau: Parents and teachers should refrain from any prerational teaching of children. MackieVsRousseau: understandable but unrealistic.

Stegmüller IV 502
Religion/Faith/Wittgenstein: Ex. if one makes a choice, the image of retaliation always appears in their mind. Meaning/Mackie/Stegmüller: one possibility: the believer wants his pronouncements to be understood literally. S_he stands by a statement of fact. But notwithstanding, such pronouncements outwardly serve to support their sense of responsibility and to justify it. Then, according to Wittgenstein, their faith would be superstition!
When asked for proof, they do not hold his pronouncements capable of truth. But then they change their position again and literally believe what they must believe.
Other possibility: faith has a literal meaning, but comparable with the plot of a novel, fiction. One can accept that the corresponding values have a meaning for life.
IV 503
Therefore we could accept that there is a God only in our practical moral reasoning. T. Z. Phillips: if the questions about God and immortality are undestood literally, as factual questions, then the skeptical response given by Hume is correct.
Thesis: one can and must interpret religious convictions and statements in a way that the criticism of Hume is irrelevant! It is true that logical and teleological proof of the existence of God cannot be upheld.
The reality of God must not be interpreted as the reality of an Object, "God" isn't the name of a single being, it refers to nothing.
IV 504
According to Phillips metaphysicians misunderstand the everyday meanings of words. MackieVs: one doesn't dissolve the real problems of skepticism by pointing to normal parlance. Just as ordinary language philosophers couldn't prevail VsHume.
Faith/Religion/Phillips: magical and religious language should be interpreted in the sense of performative actions.
Mackie pro, but: it is wrong to say that an expressive language could not at the same time be descriptive in a literal sense.
IV 504/505
Actions of faith are both: ways to address happiness and misery in the world as well as to explain them. Religion/faith/R. B. Braithwaite: thesis: the core of the Christian faith is the determination to live by the principles of morality. The "Christian stories" are accompanied by that, although the Christian is not required to believe them literally! They are religious attitudes!
PhillipsVsBraithwaite: the Grammar of "believing" and "being true" in religious convictions is not the same as in empirical statements.
MackieVs: thereby we lose any firm ground under your feet! Braithwaite rightly used the usual notions of truth and falsehood!
IV 506
MackieVsPhillips: there is no alternative to that which is discarded by Phillips, namely to continue in superstitions or to reduce religion such as that the "basic characteristics of faith are lost". MackieVsBraithwaite: certainly, numerous religious statements can be interpreted as moral attitudes, but this does not apply to the central statements of theism.
Faith/Mackie: needs an Object of reference!

Macki I
J. L. Mackie
Ethics: Inventing Right and Wrong 1977

Carnap V
W. Stegmüller
Rudolf Carnap und der Wiener Kreis
In
Hauptströmungen der Gegenwartsphilosophie Bd I, München 1987

St IV
W. Stegmüller
Hauptströmungen der Gegenwartsphilosophie Bd 4 Stuttgart 1989
Whorf, B. Black Vs Whorf, B. II 103
Whorf hypothesis/Black: there is a certain romantic enchamntment in the idea of ​​the freedom to refer to reality in different, perhaps equally valid ways, while thoughts and feelings are reflected. Language, according to Whorf, should be adapted to the needs of a nation like fish, collecting berries, etc.
II 104
Every language contains words that are particularly difficult to translate. Whorf: it was found that the linguistic system (Grammar) of each language is not only a reproductive instrument for the expression of thoughts, but rather forms those thoughts, it is a pattern and guidance for the mental activity and for the synthesis of its ideas.
The formulation of thoughts is not an independent process.
It is more or less different for different Grammars.
We structure nature along lines that are given to us by our mother tongue. (Putnam pro).
II 105
We cannot speak at all without subjecting ourselves to the order. BlackVsWhorf: that alone would not be of particular interest if Whorf had not attempted to apply this to some Native American languages.
E.g. Hopi/Whorf: the Hopi do not emphasize temporal relations, nor even the distinction between time and space, instead they have two major categories, which Whorf calls "manifest":
"Subjective"/"Objective": they are to correspond to realized or not yet realized things or to "budding" or "fermenting" things.
Reality/Hopi/Whorf: is presented as composed mainly of events. Subject and predicate are avoided.
II 106
Black: Whorf’s successors tend to quote selected suitable examples. BlackVsWhorf: for English the pendant would be that we have a strong heritage of Aristotle. With "essence", "substance", "specific", "entity", etc. But how many English speakers reflect that at all?. >Use.

Black I
Max Black
"Meaning and Intention: An Examination of Grice’s Views", New Literary History 4, (1972-1973), pp. 257-279
In
Handlung, Kommunikation, Bedeutung, G. Meggle (Hg) Frankfurt/M 1979

Black II
M. Black
The Labyrinth of Language, New York/London 1978
German Edition:
Sprache. Eine Einführung in die Linguistik München 1973

Black III
M. Black
The Prevalence of Humbug Ithaca/London 1983

Black IV
Max Black
"The Semantic Definition of Truth", Analysis 8 (1948) pp. 49-63
In
Truth and Meaning, Paul Horwich Aldershot 1994

The author or concept searched is found in the following 3 theses of the more related field of specialization.
Disputed term/author/ism Author
Entry
Reference
Names Geach, P. I 163
Subject / object / name / grammar / logic / meaning / Geach: there is a contemporary theory that names and quasi names could be no grammatical subjects. (GeachVs).   Supposedly they possessed no inherent meaning and are no words of a language!
  GeachVs: if, for example, the name "Arthur" has no meaning, then it can be neither clear nor ambiguously used!
Wittgenstein I 190
Wittgenstein s thesis: "Phenomenology is Grammar" - phenomenological analysis is the same as concept analysis. "The grammar takes the place of phenomenology".
  I 192
Thesis: that both phenomenological entities mentioned by him are secondary in relation to the physical objects.
Ontology Wright, Cr. Field I 163
... ontological inflationism: thesis: the surface grammar of sentences like "line c1 is parallel to line c2": Ontological Inflationism/Field: the obvious explanation is that Wright represents the ontological inflationism and assumes that all facts contain abstract Objects so that (a) is made true by facts that are as concrete as possible.
I 165
Def extended ontological reductionism/eoR: thesis: not only apparent singular term for directions but also existential quantification about directions are semantically misleading.
WrightVseoR.

Field I
H. Field
Realism, Mathematics and Modality Oxford New York 1989

Field IV
Hartry Field
"Realism and Relativism", The Journal of Philosophy, 76 (1982), pp. 553-67
In
Theories of Truth, Paul Horwich Aldershot 1994